Этого треда уже нет.
Это копия, сохраненная 17 сентября 2019 года.

Скачать тред: только с превью, с превью и прикрепленными файлами.
Второй вариант может долго скачиваться. Файлы будут только в живых или недавно утонувших тредах. Подробнее

Если вам полезен архив М.Двача, пожертвуйте на оплату сервера.
Тред тупых вопросов №107 MESSIER EDITION 484908 В конец треда | Веб
Тред вопросов о жизни, Вселенной и всём таком.

Спрашиваем то, за что в других местах выдают путёвку в биореактор. Здесь анонимные учёные мирового уровня критически рассмотрят любые гениальные идеи и нарисованные в Paint схемы.

Предыдущий тут: >>480719 (OP)
https://2ch.hk/spc/res/480719.html (М)

Q: Можно быстрее?
A: Можно упасть в пузырь альбукерке, наса уже почти надула его.

Q: Я начитался охуительных историй про уфологию, че делать, нам жопа?
A: Да, тебе жопа, можешь сгонять в зогач или куда оттуда пошлют.

Q: Что будет с человеком в вакууме без скафандра / если он упадет на черную дыру / попробует ступить на поверхность газового гиганта/солнца?
A: Он умрёт.

Q: Почему бы не привязать ракету к воздушному шару или стартовать с горы?
A: Космос - это не как высоко, а как быстро, большая часть энергии ракеты уходит на разгон вбок.
Подробнее тут https://what-if.xkcd.com/58/ (английский) https://chtoes.li/orbital-speed/ (перевод)
2 484912
Ебанный стыд...
Во-первых, Алькубьерре.
Во-вторых, не упасть, а создавать вокруг корабля изнутри (иначе кина не будет).
В-третьих, НАСА искривляет пространство на десятимиллионную часть, контролируя это сверхточными интерферометрами, до самого варп-привода здесь - как до Антарктиды раком.
3 484917
>>4912
Когда эту пасту уже интегрируют в шапку?
4 484918
>>4917
Ее суть как раз в том, что она ответ на шапку.
Была еще одна паста отвечающая на эту пасту, и краткожившая паста ответ на пасту отвечавшую на пасту.
5 484919
>>4908 (OP)

>M87zoom.webm


А почему это очко саурона вейпает только в одну сторону?
6 484920
>>4908 (OP)
Для искусственной гравитации на космическом корабле нужно открыть (выделить) гравитон и направить гравитоны на пол корабля?
8 484922
>>4920
Пока неясно, есть ли частицы отвечающие за силу притяжения, есть мнение, что это суть метрики пространства-времени, ее искажение.
В пользу этого говорит принцип эквивалентности ускорения.
Так что отвечая на твой вопрос: можно вообще ничего не выделять и просто ускоряться.
А если солярки мало - сделай центрифугу.
9 484923
>>4921
Ааа, из-за скорости плюм в другую сторону светит в другом спектре получается?
10 484924
>>4922
То есть достаточно сделать в космосе эффект центробежных сил?
11 484925
>>4919
Почему только в одну? В другую сторону хуярит не менее мощный джет, просто его с Земли не видно.

Из-за релятивистских эффектов движущееся в нашу сторону вещество кажется более ярким, а удаляющееся от нас — более тусклым, в случае с М87 просто разница в яркости очень большая из-за околосветовой скорости струи и направленности ее практически на Землю.
12 484930
Анон, посоветуй сайфая годного (сериалы, кинцо, книги, похуй). Главный критерий, чтобы все было реалистично (орбитальная механика, физика, технологии, мир и т. д.), но не слишком камерное, чтобы хотя бы в больших масштабах, чем лагерь на марсе.
13 484931
>>4930
Игры в этом разделе и так обсуждаются (кроме инженеров, они не про механику, это просто копрокубач): KSP, Space Engine, Orbiter
Кинцо: Apollo 13, First Man, Gravity... дополните дальше, в голову не приходит
Книги: дохуища. Лезешь в любимую библиотеку и выбираешь жанр Hard Sci-Fi, после чего выбираешь понравившуюся завязку.
Анимцо Planetes, кстати, грамотно космос показали. Если не против еще драмы-отношача заодно навернуть - понравится.
14 484932
>>4930
Почему не в sf? Средний обыватель там не до конца в состоянии оценить реалистичность мира. Нужна рекомендация ученых спейсачеров.
15 484933
>>4931
Спасибо, анон.
16 484950
Если бы наша планета была как Кербин (3 тысячи дельты до орбиты при той же массе), кто бы первый добрался до космоса? Нацисты во вторую мировую, ребята типа Годдарда в начале XX века, китайцы в средние века?
17 484954
>>4950
Очень трудно рассуждать в сослагательном ключе, особенно учитывая что например у тех же римлян были все возможности и потребности запилить телеграфную сеть, и они даже знали о светосигнальных башнях от греков, но так и не запилили.

Правильный ответ - да хуй его знает, как поперло бы так и вышло бы.
18 484955
чё ам по плоскостям?))))))
19 484972
>>4950
Думаю ребята типа Годдарда, так как для создания орбитальных ракет не требовались бы серьезные технологические достижения и индустриальные мощности
20 484974
>>484958

Где-то проебался с порядками, должно было получиться 180 раз, а не 1800.
21 484993
>>4950
Что за дельта, нахуй? Поясните.
22 485001
>>4993
Характеристическая скорость/Δv/дельта-в. Максимальная скорость, до которой может разогнаться реактивный аппарат в вакууме при отсутствии других сил, от полного бака до пустого. Она же - универсальная мера доступной энергетики аппарата, независимо от его размеров. А также универсальная мера стоимости любого перехода между двумя орбитами, независимо от характеристик аппарата. Напрямую связана с мощностью двигателя и его удельным импульсом (экономичностью).

Это базовая хуйня в спейсаче. Если такие вопросы приходится задавать, читни любой учебник по орбитальной механике (Левантовского например), поиграй в любой симулятор - KSP, Orbiter, или вообще GMAT.
24 485036
>>5034
Алё, это тред тупых вопросов, он для этого и нужен.
25 485037
>>4931

>Если не против еще драмы-отношача заодно навернуть - понравится.


Манга в этом деле лучше, там романтики нет.
15237172676340.jpg36 Кб, 374x370
26 485042
>>5034
Поссал тебе в ротеш, пидор

Автор вопроса про всплывающие пельмени
27 485048
>>5034
Анон на спейсаче жесток. Сначала посылает в ТТВ, а оттуда посылает нахуй.
28 485100
>>484958
А, так ты про градиент хотел сказать, так бы и сказал.
29 485110
Почему не юзают ГПВРД при выводе космических аппаратов на низкую орбиту?
30 485112
>>5110
Потому что твоя мамаша шлюха.
31 485157
>>5110
Потому что их не существует в юзабельном виде.
32 485174
>>5110
Потому что проскочить атмосферу на порядок проще, чем ебаться с ней. Летать на гиперзвуке тяжело, греется пиздос. Кроме того, устойчивый гиперзвуковой полет это область малоисследованная, и востребованная в основном вояками. Входить в плотные слои на большой скорости под большими улами атаки гораздо легче, чем взлетать в них на этой скорости.

А так - есть полтора маняпроекта. Скайлон, которому скоро уже лет 35 будет как он на бумаге. И какой-то китайский стартапер с сумрачной вундервафлей, глянь в китаетреде. У Скайлона есть лишь концепт движка, за гиперзвуковую тушку они даже и не брались. У китайского школостартапера якобы какая-то хитровыебанная аэротермодинамическая схема, про которую нихуя не понятно.

Алсо, движком скайлона интересовалась дарпа и вроде как года 4 назад что-то там финансировали помимо viability study. Может у них там полноценный блэк проджект, а может хуй с маслом.
33 485178
>>5174

>Алсо, движком скайлона интересовалась дарпа и вроде как года 4 назад что-то там финансировали


Вот вы тут сидите, а пиндосы скоро втихаря колонизируют Альдебаран и ни с кем не поделятся.
34 485180
>>5178
Шутки шутками, а воякам наверно такое гибридное двигло очень кстати. Всякие летающие стелс табуретки в блэк режиме и делались и эксплуатировались в свое время. А сейчас вояки трех стран спят и видят гиперзвуковые ёбы, и гиперзвуковые тушки пока что существуют только у них, так что дарпа вполне могла что-нибудь нахимичить в очередном сканк ворксе. а могла и хуй пососать
35 485184
>>5174
Если я правильно понял, сейбр в ближайшие полтора года будет проходить полноценные испытания вместе с прекулером, новость на сайте esa была, показывали строительство тестовой площадки
36 485194
А можно ли солярку в качестве ракетного топлива?
Что будет, что изменится?
А бензин?
А СПГ?
...
А МАЗУТ?
37 485226
>>5194
А можно колеса квадратные? А ромбические? А треугольные?... А можно из обедненного урана?
tenor (2).gif164 Кб, 789x258
38 485227
>>5226

>А можно колеса квадратные?


Да, если ездить по специально спроектированной дороге рилейтед.

>А ромбические? А треугольные?


Абналогично, с поправкой на колею.

>А можно из обедненного урана?


Можно, но непрактично - уран хрупкий и тяжелый, при езде по кочкам будет трясти и колеса развалятся.

А теперь на мой ответ давай.
39 485228
>>5227
Можно, можно, можно, можно. Ракета полетит. Изменится конструкция двигателя и удельный импульс. Была серия в разрушителях мифов, где они в качестве топлива использовали какое-то совсем полное говно.
40 485229
>>5228
За ответ, конечно, спасибо, но он не настолько подробный, как хотелось бы.
41 485234
>>5194
Обычные соляра, бензин, тем более мазут - вряд ли, нужна особо высокая степень очистки для фракций, иначе получится неравномерное горение и неизбежный бабах. Ракетные движки гораздо более привередливы к чистоте, чем воздушно-реактивные, а те гораздо более чем поршневые.

СПГ то же самое, хотя его, по-моему, проще очистить и получить метан например.

Всячески рекомендую читнуть Ignition! Джона Кларка
https://archive.org/details/ignition_201612/page/n11
Там из первых уст описан процесс ебли с подбором и очисткой углеводородов для ракетных движков, в красках, фаерболах и языках пламени
42 485235
>>5234

>Ракетные движки гораздо более привередливы к чистоте, чем воздушно-реактивные, а те гораздо более чем поршневые.


А я чето слышал, что газотурбинные моторы могут хоть на угле (мелкодисперсном) работать, им насрать на топливо типа.

>Всячески рекомендую читнуть Ignition! Джона Кларка


>https://archive.org/details/ignition_201612/page/n11


Благодарю за качественное пополнение в мою читалку!
43 485240
>>5235
А какая связь газотурбин и ракетных двигателей, ты к чему это
>>5194
Любую горючую хуйню можно, особенно в гибридных ракетах, вопрос только в эффективности - почти все вещества намного хуже используемых, а те что лучше фтор, бораны - нецелесообразны
44 485246
>>5240

>А какая связь газотурбин и ракетных двигателей, ты к чему это


так это одно и тоже
45 485260
>>5246
Ну нет, лол. А вообще, привередливость к чистоте горючки в ракетных берется от мощности и экономии массы. Там гигаватты могут быть, а камера сгорания и сопло подобраны в тютельку. Маленький чих у этой прорвы может оказаться такой мощности, что распидорасит все.
46 485262
>>5260
И да, особенно важен процесс запуска - при неравномерном распространении может получиться хардстарт, может так ебануть, что обломков не соберешь. Вот в Ignition!, приведенной выше, результат хардстарта буквально на первой-второй странице. и такая KSP там всю книгу
1554536442157838217.png164 Кб, 400x357
47 485266
>>5240

>Любую горючую хуйню можно, особенно в гибридных ракетах


Все опять же зависит от масштабов. Если любительская пукалка в тисках в гараже может завестись на дровах и чудом не ебануть, то практический движок обычно пидорасит нахер. Вон даже на мелком казалось бы гибриднике SS2 два или три раза топливо меняли из-за неравномерности горения. А все потому, что из-за экономии массы все подогнано в край.
48 485320
>>5266
У тебя причинная связь хромает
49 485351
Будет ли у гаусс-винтовки отдача? Пропеллента тонет.
50 485352
>>5351
а движение пули?
51 485355
>>5351
Будет.
Третий заон Ньютона.
Как пуля отталкивается от ствола, чтобы ускориться, так и ствол от пули.
52 485356
>>5352
Я знаю про третий закон и закон сохрания импульса, но не могу правильно его сюда применить. В обычном огнестреле порох по сути делает из оружия ракету, толкая всю конструецию назад плюс затвор отъезжает назад и бьет по раме. А в гавусе что будет толкать конструкцию назад? Единственное что лезет в голову - это катушки будут "отталкиваться" от пули через магнитное поле.
53 485357
>>5356
а что такое ракета? это выброс массы в одну сторону и движение в другую. пуля имеет массу, значит есть выброс массы и движение в другую сторону.
54 485359
>>5356

>это катушки будут "отталкиваться" от пули через магнитное поле


Именно так и будет.
55 485366
>>5320
Обоснуй
56 485368
>>5366
С размером связи крайне мало, двигатели делают на тщательно выверенных компонентах потому они должны работать максимально эффективно, а не потому что они большие
Ничто не мешает сделать огромных движок на говне, вопрос только в том нахуй он нужен если на нормальных компонентах тяга и импульс будут в разы выше
methalox hardstart 600fps.mp41,3 Мб, mp4,
426x190, 0:33
57 485385
>>5368
Ты похоже не умеешь воспринимать написанное. Я не про размер говорю, а про снижение массы. Ясен хуй можно сделать забетонированный демонстратор с ебически толстыми стенками, чтоб выдержал неравномерное сгорание и не взорвался на старте, но так не делают, потому что никуда он не полетит - слишком тяжелый.

>Ничто не мешает сделать огромных движок на говне


Мешает дохуя чего. Далеко не любые (даже энергетически выгодные) компоненты дают ровную реакцию, даже если они абсолютно чисты и условия идеальные. Реальный движок крайне хрупок по сравнению с прорвой энергии, которую он извлекает и направляет в нужную сторону. Небольшое отклонение от накатанной, в результате неправильно рассчитанной динамики, или неподходящих примесей, и пизда. Шебмрилейтед например вместо стандартной тонны тяги получил резкий удар в три тонны, как думаешь, на пользу это ему пошло? Это малюсенький хардстарт, на заре проектирования ЖРД, когда ошибки были в порядке вещей, это чаще выглядело как распидорас всего стенда до основания.

Я не зря Ignition! выше советовал, там от первого лица автор повествует про несколько десятилетий подбора компонент топлива, про скорость сгорания, про стабильность реакции, про постоянные бабахи которые происходили из-за этой хуйни и уносили с собой создателей, про то как создавались марки реактивных углеводородных топлив и очистку фракций, ибо ни бензин, ни обычный керосин, ни дизтопливо не подходили. Компоненты есть и получше по энергетике, но остановились на применяемых по разным другим соображениям, включая скорость реакции и ровность горения (а не только удобство обращения с ними, плотность или хранимость).
58 485386
>>5385
Я знал что на этом видео будет с первых секунд. Очень классное показательное видео.
Есть моар подобного, наглядного?
изображение.png26 Кб, 613x535
59 485389
Господа, я дико извиняюсь, но что за хуйня?
Ебашу значит в orbiter 2016, втыкаю в transx, пытаюсь слетать к марсу. После отлета от земли, в тот момент когда удаляется первый стейдж, transx перестает рисовать какую либо графику вообще. Пробовал перезапускать игру, не помогло.
ЧЯДНТ?
60 485391
>>5389
У нас же где-то был Орбитер-тред? В каталоге не найти почему-то.
111112.png915 Кб, 1221x688
61 485394
Подскажите это блик или созвездие какое, али еще что-то?
62 485395
>>5389 >>5391
Был, но настолько мертвый что потонул даже тут.
Банальная наводка - а у тебя версии-то совпадают? транс-экса и орбитера.
sage 63 485397
>>5394
1. если в высокой широте типа авроры может быть?
2. может что на камере блик какой
3. пыль от станции может слетела, какой-нибудь двигатель включался?
64 485398
>>5394
Блик.
Созвездия быть не могут видны в принципе, земля же в кадре.
Это как в светлой комнате снимать в сторону ночного окна и пытаться что-то там разглядеть. Открытого окна, чтобы не бликовало.
Яркости несопоставимы, камеры такое не берут.
S036221770.jpg1,2 Мб, 4256x2832
65 485399
>>5394
Это малафья на одной из камер HDEV, там гермоконтейнер с термостатом и обычная коммерческая камера.
66 485400
А почему до сих пор можно найти камни из катархея если есть движение плит и они по идее должны были кучу раз друг под друга залазить и переварить поверхность планеты по многу раз?
67 485402
>>5385
Еще раз.
Моя позиция в том что требования к компонентам исходят исключительно из, скажем так, серьезности и функциональности ракеты, а не размеров.
68 485409
>>5395
Бля, по ходу ты прав. У меня вот этот:
TransX V3.14 for orbiter 2010
Где отосрать для 2016?
69 485410
>>5402
В таком случае ты отвечаешь на что угодно, но не на мой пост. Я всего лишь ответил на вопрос "что будет если бензин в ракетный движок залить". Скорее всего он просто ебанёт, т.к. состав автобензинов слишком плавает. Т-1, РГ-1, RP-1 специально были созданы как особо чистые авиаварианты керосина, а не потому что энергетически выгодней.

>требования к компонентам исходят исключительно из, скажем так, серьезности и функциональности ракеты


И это кстати тоже хуйня, там требований миллион.
70 485412
>>5410
А нельзя сделать чисты бензин?
Чистую солярку?
...
Чистый МАЗУТ?
71 485414
>>5412
Можно, получится авиамазут. нахуй никому не нужный
72 485415
>>5414
Проиграл с идеи авиамазута и ракеты на мазуте.
До сих пор хихикаю как даун.
73 485416
>>5409
Всё, отбой, нагуглил. Щас попробуемс.
74 485417
>>5409
В ангаре, конечно, где ж еще.
https://www.orbithangar.com/searchid.php?ID=6393

Не забудь еще соответствующую тему форума глянуть, в теле поста ссылка и внизу юзер комментс ссылка.
75 485425
>>5410

>В таком случае ты отвечаешь на что угодно, но не на мой пост


бля ну нить не теряй

>Любую горючую хуйню можно, особенно в гибридных ракетах


>Все опять же зависит от масштабов


>Я не про размер говорю, а про снижение массы. Ясен хуй можно сделать забетонированный демонстратор с ебически толстыми стенками, чтоб выдержал неравномерное сгорание и не взорвался на старте, но так не делают, потому что никуда он не полетит - слишком тяжелый.


снижение массы и прочие параметры - это следствие назначения двигателей

>Я всего лишь ответил на вопрос "что будет если бензин в ракетный движок залить". Скорее всего он просто ебанёт, т.к. состав автобензинов слишком плавает


ебанет от бензина он потому что вследствии своего назначения двигатели должны быть максимально эффективными, следовательно они должны работать на предельных параметрах. с размером и масштабами, как ты говорил, связи нет - никто же не делает мелкие орбитальные пукалки и рсу на говне,
короче хуй знает зачем ты вообще поднял тему размеров, правильный ответ должен быть таким : "создать реактивный двигатель можно на любом горючем веществе, создать эффективно работающий и безопасный двигатель можно только на некоторых чистых веществах водород-кислород или тщательно подобранных компонентах с определенным составом керосин, гидразины, петардные смеси и прочее"
76 485427
>>5425

>короче хуй знает зачем ты вообще поднял тему размеров


Не, у тебя явно проблемы с пониманием. Я её не поднимал. Я наоборот сказал, что ничего от них не зависит, см.>>5266

>даже на мелком казалось бы гибриднике


А масштабы это вообще не про размер было, а масштабы затеи.

Короче проехали, ты прицепился непонятно к чему вообще, и какую-то свою хрень гнешь
77 485428
>>5400
Постоянно возникает и переваривается только океаническая кора, сами континентальные плиты гораздо стабильнее.
78 485429
>>5428
Воу, спасибо.
А почему так?
Где можно в доступном для олигофренов сжатом виде ознакомиться с этим?
Желательно в виде научпопа на английском.
79 485430
>>5425

>создать реактивный двигатель можно на любом горючем веществе


Нет нельзя, блять!
На любом говне можно демонстратор создать, зажатый в тисках и нелетающий. Которых на ютубе у кузьмичей полно. Или ракетные санки, в которых более-менее похуй на массу, или что-то в этом духе. Чтобы ЖРД двигал ракету, даже любительскую хуевину на километр поднять, надо чтобы он легкий был и хрупкий, относительно управляемых энергий. Поэтому, кстати, почти и нет кустарных летающих ЖРД - они либо требуют довольно точной машинерии, либо перетяжелены.
80 485431
>>5430
Самая жопа же в хрупкости сопла?
Почему бы не обойтись тогда без сопла лаваля? ТНА похуй на вид топлива по идее.
81 485433
>>5427

>масштабы затеи


бля сразу бы сказал
>>5430
я не понял, почему у тебя нелетающий двигатель не двигатель?
82 485435
>>5433

>я не понял, почему у тебя нелетающий двигатель не двигатель?


Потому что так можно договориться до того, что ацетиленовый сварочник - двигатель, и фонарик тоже. фотонный
83 485436
>>5435
Ну технически...
84 485440
>>5435
Ну, вопрос в применении и идеи во время создания
Если я соберу хуйту которая сможет волочится по земле на своей тяге это же будет двигатель, не так ли?
85 485550
86 485565
Почему масконы на луне настолько сильно на орбиту влияют?
Если такая большая проблема, почему нельзя просто орбиту выше сделать?
87 485567
>>5565
масконы страшны если ты о них ничего не знаешь и выбрал орбиту наобум очень низкую. Если ты о них знаешь то мы можешь выбрать орбиту либо более высокую, либо более эллиптическую, либо выбрать тщательно выверенную frozen orbit, либо заниматься поддержкой орбиты как на МКС, следя за высотой.
88 485624
>>5565

> Почему масконы на луне настолько сильно на орбиту влияют?


Осне большая разница в плотности. Есть места на краю масконов, где вертикаль отклонена на градусы.

>Если такая большая проблема, почему нельзя просто орбиту выше сделать?


Потому что часто нужна ниже. При любой посадке это тоже проблема.
photo2019-05-1517-25-15.jpg126 Кб, 706x1022
89 485638
Поясните, что тут говорится?
https://zen.yandex.ru/media/funscience/merkurii-deistvitelno-mojet-okazatsia-samoi-blizkoi-k-zemle-planetoi-5c8f6b0de6d4d400b3addcc2
Я что-то так и не понял, почему из этой статьи выходит, что Меркурий - ближайшая к Земле планета. Какая-то подборка статистик. Я прям чувствую, что тут подвох, но в чём, никак в толк не возьму
90 485639
>>5638
Хули непонятно-то? Венера ближе всего _бывает_ к Земле, но в среднем дальше, чем Меркурий.
91 485641
>>5639
То есть, именно Меркурий нужно использовать для гравиманёвров хуиты с Земли?

Абажжи, ёбана. Но тогда получается, что и Земля к Сатурну будет ближе чем Юпитер.
Чё за бред.
92 485642
>>5641

>именно Меркурий нужно использовать для гравиманёвров хуиты с Земли


Схуяли? "Ближе" не значит, что к нему проще подлететь. Да и масса у него маленькая, а находится он глубже в гравиколодце Солнца.

> тогда получается


Не факт, подозреваю, что там эксцентриситет орбиты влияет. Орбиты планет ведь не совсем круглые.
93 485656
>>5641

> То есть, именно Меркурий нужно использовать для гравиманёвров хуиты с Земли?


С хуя ли? В чем логика такогт вывода?

> Абажжи, ёбана. Но тогда получается, что и Земля к Сатурну будет ближе чем Юпитер.


> Чё за бред.


Да, возможно, в среднем чаще к Сатурну ближе оказывается Земля, чем Юпитер.
Другое дело, что данный факт вообще ни на что не влияет. Дело в том, что в космосе все постоянно находится в движении, поэтому понятия "ближе", "дальше" в некоторых случаях не так важны, как "скорость", "масса", и т.д. И тем более когда речь идет о средних значениях. Среднее расстояние, блядь, да нахуй кому оно нужно
94 485669
Сап. Поясните за космическое производство: типа я с бухим батей спорил сегодня, зашла речь о том, что в Раше космос не мертв, и мы пендосам движки поставляем. Я смутно помню, что это типа мифа, но хотелось бы точных подтверждений (или опровержений)
95 485672
>>5669
Это не миф, Атласы летают на РД-180, Антаресы на РД-181.
Пруфусы прямо на сайтах производителей:
https://www.ulalaunch.com/rockets/atlas-v
http://www.northropgrumman.com/Capabilities/Antares/Pages/default.aspx
96 485673
>>5672
Окей. Так космос этой стране жив или как? На космических каналах в ютубе говорят, что отрасль проебали, но откуда движки тогда? Наследие совка?
97 485674
Каюсь, сразу не загуглил. Рд-180 в 90-е разработали, до 14 производили. Рд-181 не нашёл инфы на педивикии. Так ещё больше вопросов: поставки до сих пор происходят? Если да, то хуяли в 14 завершили производство. Сори за тупость: рубит уже пиздец
98 485686
>>5669 >>5673
Долго расписывать, это надо в роскосмос тред. Всё мертво, опыт давно потерян, т.к. разные области индустрии простаивают (в смысле вообще нихуя не делают) от 8 до 20 лет приблизительно, интереса к ним нет ни у государства, ни у бизнеса; в обозримом будущем (очень коротком, как обычно) тоже нихуя не предвидится. И дело не в деньгах. Исключения - ИСС и еще несколько контор, у которых всегда будет мелкий госзаказ на связь и ДЗЗ. У пилотируемой вообще 0 шансов после МКС. Но мозги нам ебать будут очень долго.
99 485687
>>5674
А мог бы зайти и на сайт производителя двигателей.
РД-181 производится с 2015. РД-180 в производстве тоже в настоящее время.
100 485717
Почему чёрные дуры сохраняют эл. заряд, хоть и недолго? А магнитное поле, например, нет.
101 485737
На орбите Земли вращается МИР (или что там сейчас вращается). Ему навстречу по этой же орбите/под углом, по прямой траектории столкновения к станции несётся небольшой астероид/метеор. Космическое тело врезается в космическую станцию. Допустим, станция сделана из суперкрутых материалов и не сломалась от такого столкновения, однако удар оказался таковым, что удар полностью нивелировал скоростью движения по орбите этой станции, сделав её нулевой относительно орбиты. Из-за этого закономерно это рухлядь начинает падать на Землю.
Собственно, вопрос: насколько быстро это груда металла упадёт с небесной тверди на земную?
Можно ли будет успеть просчитать траекторию падения?
А если \этот хлам будет падать на большой город, моно ли будет как-то спастись? Кроме бункеров и прочего ведь падение такой штуки должно мало того что сопровождать дождём из осколков, так ещё и кинетическая энергия в тротилловом эквиваленте (Мужик_с_ракеты_упал.пнг).
И ещё, если подумать логически, то помимо остановления вращения по орбите стнации, камушек-то тоже обязан остановиться, разве нет? Значит, и он падать начнёт. И в то же место, что и станция, разве нет?
То есть, гипотетический урон в подобном сценарии огромен
large.jpg21 Кб, 494x484
102 485745
103 485751
>>5737
Вопрос сформулирован подегенератски, но попробую ответить. Предположим, что их нерасколошматило в осколки (что и произошло бы), предположем каким то хуем они остановились. То есть, что, если МКС (не МИР, сука) замедлит орбитальную скорость до нуля? Высота МКС 408 км. Предположим, что воздуха нихуя нет. Тогда у нас есть волшебная формула t = корень(2h/g), где g — ускорение саободного падения. t = корень(2 408000 / 9.8) ~= 290 с. v = g t ~= 2.8 км/сек. То есть скорость приземления будет всего нихуя (как у истребителя, уебавшегося носом на полной скорости в землю. Если добавить воздух и предположить, что станция каким то хуем не развалится в воздухе, то сопротивление будет сильно заглушать ускорение и приземление при ударе максимум провалит крышу какого-нибудь дома (если очень повезёт попасть).
Короче, метеориты гораздо опаснее, потому что они уже из космоса летят в землю на неебической скорости от 11 до 72 км/сек, а вышеописанное ничего, кроме пука в лужу не сделает.
104 485752
>>5751
предположим
2 x 408000


быстрофикс
даблфейспалм1.jpg44 Кб, 600x530
105 485753
>>5737
пиздец ты тупой, даже по шкале градации тупости не найти насколько ты тупой
106 485756
>>5753
Добро пожаловать в реальный мир, анон. 99% людей вообще не ебут как работают космос и физика, но анону хватило смелости не промолчать и раз уж мы в ТТВ, то нужно быть готовым отвечать на любые вопросы. Лучше задавать тупейшие вопросы, вместо того, чтобы держать их в себе и не развиваться никак.
107 485767
>>5751
Хорошо, а если два липких тела (две кометы из клея "наномомент") врежутся в друг друга одинаковой скорости так, на противоположеной по вектору, упадёт ли их совместная пост-ударная скорость настолько, что они упадут на Землю?
За сколько они упадут?
Насколько разрушительной будет кинетическая энергия от падения?
108 485768
>>5767
Все выше применимо и здесь. Будет пук в лужу.
Non Cerebri - Non Forsit.png37 Кб, 640x480
109 485771
>>5756
Раз такая пляска, может объясните мне, что такого взрывного в том посте?
110 485786
>>5751
Сколько там весит мкс? 200 тонн? Кинетическая энергия=200000 2800 2800 / 2 = 7.84 10^11 Дж
Метеорит массой 1 тонна и скорость 30 км/с = 1000
30000 30000 / 2 = 4.5 10^11 дж
1506913491056.png97 Кб, 397x189
111 485792
>>5737
Что за хуйню я читаю? Пиздец, какой-то наркоманский бред.

Но ладно, если свести твой вопрос к "что будет, если на орбите лоб в лоб столкнутся два одинаковых идеальных объекта с одинаковой скоростью", то

>насколько быстро


Зависит от высоты, а ускорение свободного падения известно, задача для 5 класса средней школы. Атмосфера затормозит прилично, немного отклонит друг от друга. С низкой орбиты - несколько минут, результирующая скорость будет не очень большая.
112 485794
>>5792
А спрогнозировать место падения можно будет успеть до падения?
Ведь ещё и планета крутится и от удара траектория может сместиться
113 485796
>>4908 (OP)

>Можно упасть в пузырь альбукерке, наса уже почти надула его.



Может ли расширение пространства толкать, а точнее создавать силу, что будет толкать тело - упорядоченную группу атомов, а не прост расширять пространство и расстояние между атомами?
114 485797
>>5794

>А спрогнозировать место падения можно будет успеть до падения?


Да
115 485812
>>5786
Окей, давай добавим сопротивление и посчитаем терминальную скорость. Выйдет не больше ста метров в секунду.
116 485813
>>5812
Ну не сто. Зависит от объекта. Астероид побольше, чудом не разрушившаяся бочка или обломки - поменьше
мимо
117 485938
>>5786
МКС весит 400 ~тонн
Челябинский метеорит весил больше 9000 тонн.
pic4d4e5a844d74840394381cc45bb9f26f[1].jpg55 Кб, 420x280
118 485952
>>5938
шта?
120 485967
>>5771
Во-первых, пост уже начинается весело. Я не знаю в какой жопе мира надо жить, чтобы не знать что сейчас на орбите МКС, а Мир давно почил в глубине Тихого океана.

Во-вторых

>Ему навстречу по этой же орбите/под углом, по прямой траектории


Таки по этой орбите, под углом, или по прямой? Из контекста становится понятно, что автор имел ввиду, что вектор скорости астероида должен быть направлен в противоположную сторону в момент столкновения.

3. Зачем вообще приплетать сюда станцию, если она не разрушается? Автор мог бы написать "два тела движутся навстречу друг другу". А, но станция, по словам автора вопроса, разрушается в атмосфере, поэтому нужно спасаться от дождя из осколков.

4. Первая космическая = 7.9 км/с. МКС движется чуть медленнее. Если она полностью остановится, то ее скорость будет равна 0. Дальше - задача из школьного учебника.

5. В общем, переформулирую вопрос автора, как его бы задал нормальный человек:

Что будет, если в МКС врежется другое тело таким образом, что скорость обоих тел упадет до нуля, но станция не разрушится. Каковы последствия падения этих двух тел на Землю?

Я не часто здесь пощу, только бы не проебаться, я не хочу чтобы меня стебали в ТТВ
1558967701111.jpg47 Кб, 575x385
121 486013
Как сделали это фото?
122 486015
>>6013
при помощи камеры
123 486018
>>6015
А камера где располагалась?
инб4 в космосе
На каком космическом аппарате была эта камера?
124 486019
>>6018
Я ебу? Я тебе должен расследование проводить на тему того какие аппараты это, в каком году, где ты взял это фото? А ты хули делать будешь?
125 486020
>>6019
Хорошие вопросы.

>Я ебу?


Судя по реакции - нет.

>Я тебе должен расследование проводить на тему того какие аппараты это, в каком году, где ты взял это фото?


Было бы неплохо, если бы провел. Орбитальных станций не так и много.

>А ты хули делать будешь?


Мастурбировать и играть в игры.

Спасибо за вопросы, задавай еще.
126 486022
>>6019
>>6015
Нахуй ты высираешься здесь если ты даже не в курсе когда была сделана эта фотография ебанько?
И вообще это не совсем фотография, а кадр видеозаписи.
127 486023
Такие же есть и с мкс. Только как их делают? Через телескоп?
128 486028
>>6023
К космическим станциям обычно летают что? Правильно, космические же корабли.
Если то был кадр с видеозаписи, то мог быть Прогресс. Или огурцы из Союза сфоткали. Что, обычно, и делается.
129 486034
>>6022

>если ты даже не в курсе когда была сделана эта фотография


То есть вопрос был адресован тому, кто знает наизусть видео всех космических станций и может опознать источник видео всего лишь по одному кадру?
130 486101
>>4908 (OP)
Как так получилось, что более чем полвека назад люди на Луну летали, а сейчас проблема отправить человека на орбиту Земли?
131 486110
Тут написано https://zen.yandex.ru/media/murkosmosa/prislannye-snimki-mars-express-podtverjdaiut-chto-na-marse-est-voda-v-jidkom-sostoianii-foto-5ce1925883178c00b335fd0a что на Марсе нашли жидкую воду. Что думаете?
132 486112
>>6101
Где проблема?
Щас аппараты к марсу летают,к сатурну, мягко садятся на астероиды и кометы
Щас охуеть как продвинулась космонавтика, а человек в космосе просто юзлес, вот и не пускают
Хотя на мкс часто летают люди, а на луну в 2024 и так наса отправляют людей
133 486124
>>6101
Так на луну для флаговтыка и чтобы ДАТЬ ПОСОСАТЬ летали, тратя миллиарды нефти. А сейчас уже не нужно выебываться перед другой страной.
134 486131
>>6110

>Что думаете?


Не стоит в спейсач тащить статьи из непрофильной мурзилки, вот что я думаю. Ни ссылок на оригинальную работу, нихуя, один кликбейт и перевирание.

Но ладно, давай разберем, дело в том что озеро это нашли еще год назад по профилям радара MARSIS с аппарата Mars Express, датированным 2012-2015, вот статья
https://dx.doi.org/10.1126/science.aar7268 , пикчи в твоей мурзилке оттуда, смысл тоже. Может речь о том, что подтвердили другим методом или нашли другое озеро, но новых работ по этой теме я не вижу ни в Icarus, ни в других планетологических журналах, так я как профессиональный телепат мирового уровня склоняюсь к тому что они просто переврали прошлогоднюю статью и выдали за новую.

Вот поэтому не стоит тащить сюда всякое говно из невнятных источников, а то будем как мартышки обсуждать какую-то виртуальную хуйню без связи с тем что реально произошло.

Касаемо воды на Марсе - ну збс чо. Если подтвердят другим методом и к результатам будет больше доверия а может даже и так, это будет один из первейших кандидатов на исследование с поверхности, через АМС или даже космолюдьми, если таковые будут на Марсе когда-нибудь. В любом случае, какие-либо подробности или опровержения будут через годы.
135 486132
>>6131
Алсо, так-то есть отдельный марсотред >>481688 (OP)
136 486157
>>6101
Нет проблемы. Нет бабок на это. Потому что нахуй никому не надо.
137 486167
>>6112

>Где проблема?


Американцы например уже сколько лет не летают. В рахе тоже никак новый корабль построить не могут. Раньше в 57-ом году первый спутник, в 61-ом первый человек в космосе, в 69-ом уже на Луне. А сейчас у Маска какая-то неполадка с кораблем и уже неизвестно на сколько лет пилотируемый запуск откладывается, не говоря уже о том сколько лет его до этого делали.
>>6124

>тратя миллиарды нефти.


Ну первый раз еще можно понять все затраты и трудности, но когда уже есть опыт, новые материалы, вычислительная техника и вообще прогресс во всех областях это как-то странно.
138 486171
>>6101
В том же, в чем и у остальных - застой в космической отрасли после спадения пропагандистской и милитари нагрузки времен холодной войны. Любая организация рано или поздно замедляется, любой процесс останавливается, нужны конкуренция и приход новых, чего почти нет. Пейсх слегка расшевелили болото, делая довольно банальные по меркам космических гонок вещи. Вторая причина - раньше это делали усилиями целых стран, сейчас никто таких денег тратить не будет, тем более что на какие-то принципиально новые вехи за сжатые сроки нужны принципиально новые суммы, по сравнению с которыми даже аполлон - возня в песочнице. Нужен нормальный бизнес, а пилотируемая космонавтика бизнесу не особо интересна.
139 486172
>>6167

>но когда уже есть опыт, новые материалы, вычислительная техника и вообще прогресс во всех областях это как-то странно


Весь этот прогресс не отменяет принципиальной сложности пилотируемой космонавтики (сложность не в смысле "трудно", а "заморочено"), а это и есть основной источник её стоимости.

Ну и risk aversion, да. Регулярная доставка огурцов в космос еще не перевалила за критическую частоту, а безопасность хотят уже сейчас. В результате не получают ни того, ни другого, получая уловку-22.
140 486192
>>6167
Посмотри сколько бабла в космос вливалось тогда и сейчас.
Вот и всё. Одна единственная причина.
141 486222
>>6167

>Раньше в 57-ом году первый спутник, в 61-ом первый человек в космосе, в 69-ом уже на Луне


Ну так между первыми европейцами в Америке и первыми европейскими колониями также широкий исторический перерыв присутствует.
142 486279
Как на Марсе с аккустикой? Если я буду на Марсе, сниму шлем и попытаюсь докричаться до другого человека, который снял шлем, но стоит на расстоянии в сто метров, то он меня не услшит, как бы сильно я не кричал?
143 486280
>>6279

>с аккустикой


С этим даже на Земле хуево.
144 486281
>>6280
ЧТО?
145 486288
С чем связано увеличение скорости этой борды? Некоторые треды висели годами, но теперь, по крайней мере 4 треда уже давно номерные. Скорость борды с каждой неделей растёт, хотя космических запусков не особо больше становится.
146 486289
>>6288
Кербанов вернули в раздел
147 486291
>>6288
Модеры перестали банить за каждый пук не по правилам.
148 486298
>>6288
Нет никакого увеличения. Просто шитпостинг сконцентрировался в паре тредов, остальное никому не интересно, как это было до превращения доски в /б/ во времена Розетты
149 486300
>>6291
Единственная адекватная инициатива. А то как в гулаге, по 2 поста в неделю с этими ебанутыми банами ни за что, номерными тредами и вонючими жуками.
150 486304
>>6300
Не, ну про жуков ты зря. Это ж икона доски!
151 486308
>>6300
Лучше 2 поста про космонавтику, чем 200 про рогозина и грымзу
152 486309
>>6308
Насколько я помню, во времена адовой анальной модерации 2 поста в неделю так же были про рогозина и грымзу.
153 486345
Как психологически готовят космонавтов?
Какое направление предпочтительнее?
154 486346
И ещё, какие именно лекарства есть в космосе на текущий момент ?
Из списка ноотропов, стимуляторов, транков, витаминов и т.п
И ещё, я слышал раньше и сейчас в космос берут такой препарат как фенибут, хотя он не доказал какой либо эффективность в плацебо контроллируемых тестах. Как так? Или такое возможно только на российской стороне?
Какая аптечка в сша?
155 486365
>>6300

>вонючими жуками


Пиздец, старейший тред на доске а может и на всем мейлаче, а он нос воротит.
156 486369
>>486350
>>6349 (Del)
Пидораха, ты не в /b/, если что.
157 486370
>>6345

> Как психологически готовят космонавтов?


- во-первых, отсеивают психов из кандидатов
- дроч ситуаций до автоматизма и хладнокровия - в имитаторе капсулы с пожаром на борту, в бассейне для ВКД и т.д. и т.п.
- вместе живут в бочке некоторое время, чтоб в реальном полете не перегрызлись (что все равно случается иногда)
- общее обучение принятию решений в условиях непоняток, ограничений, дезориентации
и естественно непрерывное наблюдение и снятие метрик

>И ещё, я слышал раньше и сейчас в космос берут такой препарат как фенибут, хотя он не доказал какой либо эффективность в плацебо контроллируемых тестах. Как так? Или такое возможно только на российской стороне?


Не в курсе, но не удивлюсь если это так, с этим всегда были и есть проблемы.
158 486385
>>6365
На 4 года старше:
https://2ch.hk/tr/res/33.html#bottom (М)
159 486386
>>6370

> ну тип готовят хуё моё в бочке сидят учатся там тип хуй знает))))


Я вроде конкретно спрашивал, какой подход исповедуют в наса
160 486408
>>6385
Ну тогда на этой доске как минимум.
161 486478
Что подразумевал Кеннеди под "do the other things"?
sage 162 486482
>>6478

>We choose to go to the moon in this decade and do the other things, not because they are easy, but because they are hard


Науку на Луне, космические станции, межпланетные зонды? Смотри в контексте того времени.
163 486520
Как масса влияет на пространство?
164 486526
>>6520
Да.
165 486528
>>6520
Тебя это ебать не должно
166 486530
>>6291
Раньше моча лютовала дико. Я как-то словил бан за невинный вопрос, о том что будет, если космонавт, пролетая над горизонтом событий, сунет хуй за горизонт.
167 486531
>>6530
Может бан и был ответом на твой вопрос, глупенький.
168 486532
>>6530

>Раньше моча


Все логично, нет космоса - нет мочи.
Даже лахта забила на мертвый тред мертвой рогозы - поэтому моча уже не нужна.
169 486533
>>6526
Волынская резня
170 486535
>>6532
Иногда даже интересно, что за хуйня происходит в голове у людей, которые настолько зациклены на порашной повестке. Так и представляю, как такие утром случайно ударившись пальцем о быльце кровати или пролив на себя кофе кричат на всю квартиру АХ ТЫ ЁБАНЫЙ ПРОКЛЯТЫЙ РОГОЗИН/МАСК/ПОЛИТИЧЕСКИЙ ДЕЯТЕЛЬ США ИЛИ РОССИИ! и идут писать на спейсач про лахту или русофобов.
171 486539
>>6535
Лахта на месте, я спокоен.
172 486546
>>6533
Бензопилой?
173 486568
Почему интерферометрию делать в видимом спектре тяжелее чем в микроволнах?
Я хочу задействовать Кек и Очень Большой, например.
Почему нельзя?
174 486571
>>6346
Спроси Артемьева, он вроде в соцсетях более-менее активный до сих пор.
175 486576
>>6386

>Я вроде конкретно спрашивал, какой подход исповедуют в наса


Вроде? А может не вроде?
>>6345

>Как психологически готовят космонавтов?


>Какое направление предпочтительнее?



Энивей, в наса такой же подход. Какого конкретно ответа ты хотел?
176 486577
>>6568
Потому что длина волны меньше. Отсюда резко возрастают требования к определению расстояний между приёмниками, к джиттеру линии связи и точному определению задержек, к вычислительным мощностям сведения всей этой хуйни в картинку.
177 486579
>>6386
Нихуя ньюфаги охуевшие пошли.

>тупой вопрос хуйпойми о чем


>>обзорный ответ


>ррряяяя недостаточно подробно


>и воще я не то спрашевал

178 486581
Почему не сделают хотя бы маленькую пусковую петлю 50-100км примерно, чтоб с ускорением 100-300жэ ништяки забрасывать? Митолы, воду. Построить робозавод, клепать звездолёты прямо там.
179 486657
>>6581
Дорого.
180 486659
>>6581
Пора уже в шапку добавлять.
181 486660
Почему когда Гагарин летал ему не дали кинокамеру или фотоаппарат хотя бы? Ведь эпохальное событие же.
182 486669
>>6660
Ну почему, там была SSTV-телекамера К-100, снимавшая его, она же и на пленку фиксировала. Но качество было пиздос. Вообще, у совков всегда было туго с фото-видео-регистрацией. Дело даже не в элементной базе и не в секретности, просто сама идея что-то снимать на борту/с борта, а потом показывать - была непонятна. Даже эту камеру Гагарину запихнули не чтобы запечатлеть эпохальное событие, а чтобы следить за его состоянием. Из-за этого неприятия, и из-за пренебрежения вояками и индустрией аэрофотосъемкой наработки 40-х по оптике (которые были неплохи на то время + захваченные в Германии технологии) не особо развивались, в результате чего всю космическую эру совок сосал по фото, кино и видео аппаратуре, да и потом. А аналога Швеции, в которой Хассельблад делал лунные фотики, в варшавском блоке не было, и пленка хуевая была. Всё ДЗЗ опять же использовало сильно перетяжеленную оптику из-за этого. Даже блять захваченную в середине 50-х спецпленку с одного из американских шпионских стратостатов Genetrix использовали потом в станции Луна-3, ибо в совке такой не делали.
tv1.jpg47 Кб, 730x420
183 486670
>>6660
Во-первых, какие-то телевизионные камеры внутри были, которые самого Гагарина снимали.

Во-вторых, а нахуя ему камеру было давать? Что бы он с ней делал, свои ноги снимал? Гагарин мало того что весь полет лежал пристегнутым в кресле, так еще и катапультировался из капсулы Востока и приземлился на парашюте, с собой он ничего из капсулы забрать не мог, все об землю ебнулось.
184 486671
>>6670

>Что бы он с ней делал, свои ноги снимал?


Мог бы Землю в илюминатор снимать.
185 486705
>>6670
Технически-то вполне решаемо, хоть и заморочки были бы. Например можно было бы на кресло захуярить фотоаппарат, а в кабину поставить зеркало. Чтобы не нужно было большое@тяжелое зеркало - сделать его чуть выпуклым, чтобы как на ширик получалось, а объектив длиннофокусный. Другое дело, что внутренности передового КА в газете показывать - это же гроб гроб режим кладбище враг подсмотрит, а обложка Науки и Жизни №5/1961 обошлась бы и иллюстрацией, по их мнению, чай не Лайф какой-нибудь загнивающий и не буржуазное насо, которое изначально придерживалось почти полной публичности.
186 486709
>>6670
Камеру с собой в корабль, широкоугольный объектив, можно крючок и спусковой тросик. Пленку разрядить и в карман. Меркури вообще надевали на себя, но малый объем не стал помехой для селфи космочеловека в летающем гробу.
187 486778
>>6669

>Даже блять захваченную в середине 50-х спецпленку с одного из американских шпионских стратостатов Genetrix использовали потом в станции Луна-3, ибо в совке такой не делали.


Вот все про это говорят как про очевидный факт, но я ни разу пруфсов не видел, зато видел отсутствие упоминания этого у Чертока когда он про Луны писал.
188 486789
>>6778
Оригинальный источник этой кулстори - журнал НК за август 2000, инфа из первых рук кого-то из дидов.

>зато видел отсутствие упоминания этого у Чертока когда он про Луны писал


Да у Чертока дохрена чего нет, это ж не энциклопедия. Каманина того же почитать, там тоже то чего у него нет. А вообще если ты хочешь прямо историчности - мемуары самый ненадежный источник, и в исторической науке всерьез их никто не воспринимает. Так что дохуя баек, и эта, и чертоковские, и каманинские, могут оказаться пиздежом или приукрашенными, или перевранными, в конце концов. Имеем то что имеем.
189 486830
>>6669

>Даже блять захваченную в середине 50-х спецпленку с одного из американских шпионских стратостатов Genetrix использовали потом в станции Луна-3, ибо в совке такой не делали.


зачем использовать пленку если она все равно не вернется на землю и фоток с нее не сделать?
190 486832
>>6830

>зачем использовать пленку если она все равно не вернется на землю и фоток с нее не сделать?


Пленку потом медленно сканировали используя достаточно мощную лампочку и это уже отсылали.
191 486848
Горячо ли в центре Плутона и Тритона? Хотя там может быть горячо только из-за распада радиоэлементов, есть ли там горячая мантия ещё?
192 486852
Почему большие обсерватории делают один большой телескоп вместо сотни маленьких? Зеркалами соединить изображение со всех телескопов в одну точку и готово, или это хитрый план по попилу бабла?
eelttelescopepictureoftheday31366.jpg282 Кб, 1366x657
193 486854
>>6852

>Зеркалами соединить изображение со всех телескопов в одну точку и готово


Ты не поверишь!
194 486856
>>6854

>JWST planned 2018

(Изображение JPEG, 679 × 1024 пикселов).jpg147 Кб, 679x1024
195 486889
>>4908 (OP)
прошу рассказать плюсы и минусы этого бинокля
Сам не разбираюсь в физике и астрономических наблюдениях, это первая покупка. Читал, что линзы портятся со временем, по краям плохо всё итд.
197 486903
>>6889
Это город с орбиты в бинокль?
198 486904
>>6852
Смотря что ты имеешь в виду. Если получить одну картинку с кучи зеркал - это и так делается в большинстве больших телескопов. И не только потому, что большие зеркала делать дорого и сложно доставлять, но и потому, что адаптивная оптика и подобные ухищрения по компенсации атмосферы этого требуют. Если же ты имеешь в виду сотни маленьких картинок, и затем простакать - то просто так это не получится, число картинок в стаке растет нелинейно, и нормальной чувствительности таким способом не получить, нужно именно большую апертуру.
199 486905
>>6889
>>6890
С этим лучше в телескопотред обратиться. >>429065 (OP)
СхемаСпэйсшаттла.jpg217 Кб, 1200x960
201 486962
Если грузовой бак шаттла заполнить максимально возможным количеством топлива, можно ли будет на нем слетать до Марса и назад, или даже дальше?
Проблемы радиации и жизнеобеспечения игнорируем.
202 486964
>>6962
Не. 27.5т ПН, 80т заправленный орбитер. Допустим повезет он две тонны огурцов и чипсов для них, и тонну будут весить емкости, получится 23.5 тонны вонючки для движков с УИ 316с. А получить надо как минимум 4.25км/сек только до низкой орбиты Марса, и это прям хорошие окна.
203 486965
>>6962
Да там дохуя реальности придется игнорировать, какое уж там ЖО. Этот пепелац не мог летать в принципе без затенения Землей, слишком жарился на солнце. Потому пуски были сильно ограничены. А уж тем более НОО никуда не мог из-за этого улететь.
204 486967
>>6965

>А уж тем более дальше НОО


фикус
205 486968
>>6964
А, например, до Луны и назад хватит?
206 486973
>>6968
А тебе обязательно без внешнего бака?
Если взять водород/кислород то до марса нужно взять внешний бак размером примерно(!) 1/5 от шаттловского , плюс внутри.

Так то шаттл предполагался частью STS где был ядерный буксир на NERVA, и он то ужо потом мог в марс.
207 486975
>>6973

> А тебе обязательно без внешнего бака?


Да, потому что топливо из внешнего бака полность расходуется для выхода на НОО.
208 486979
>>6975

>Да, потому что топливо из внешнего бака полность расходуется для выхода на НОО.


Тебе что ли в однопуск обязательно? Тогда да, хрен тебе ни луны ни марса. Даже ядерный буксир (если буксировать сам шаттл) не поможет.
209 486980
>>6979

> Тебе что ли в однопуск обязательно?


Да, моя фантазия основана на этом.

> Тогда да, хрен тебе ни луны ни марса.


Разве 20 тонн топлива не хватит слетать до Луны?
210 486987
>>6980

>Разве 20 тонн топлива не хватит слетать до Луны?


Неправильная постановка вопроса, надо про дельту интересоваться, т.к. "количество топлива" бессмысленно без удельного импульса и сухой массы. А вообще до низкой лунной орбиты примерно столько же нужно, сколько и до марсианской, чуть поменьше.
211 487022
>>6980

>> Тогда да, хрен тебе ни луны ни марса.


>Разве 20 тонн топлива не хватит слетать до Луны?


Не хватит.

Для того что бы на 20 тоннах топлива на шаттле слетать до луны нужен двигатель более эффективный чем нерва2.0 (тимберлейк) который рисовали в маня 80-х, а тот был тот еще маня-огого.

Но в два пуска (40тонн "топлива" условно) на луну норм. В три пуска на марс норм. Без возврата. Ну и водород все равно столько не хранится даже сейчас.
212 487023
>>7022
Да, это все естественно для NTR (ядерной тепловой ракеты), проект нерва-тимберлейк, вся фигня
213 487024
>>6987

>т.к. "количество топлива" бессмысленно без удельного импульса и сухой массы.


Он дал начальные условия норм вообщето - летим на спейсшаттл орбитере, используя грузовой отсек для доп топлива и оборудования.
214 487026
>>7022
Собственно продолжая бред - сначала один пуск ДелтаХеви, водород храним в доработаном центавре(верхняя ступень). хз могут понадобится доп резервуары для водорода Потом запускаем шаттл с ядреным движком. Шаттл стыкуется с центавром, ядерный движок поднимается из грузового отсека и жжет тягой. Центавр с доп баками после израсходования водорода отцепляем.
215 487029
Раз до Луны не долететь то куда максимально можно, может то ГСО?
216 487030
>>7029
Спечётся он на ГСО от солнышка. Да и хули там делать? Радиационный курорт для туристов устраивать? Это ж прямо посередине внешнего радиационного пояса.
217 487031
>>7030
Печальную реальность оставим за скобками, меня интересует теоретическая возможность.
218 487033
>>7031
Ну теоретически-то с баками в трюме у него хватит дельты до ГСО, да.
iTdoM3Fe0nc.jpg122 Кб, 800x800
219 487145
Возможно ли объяснить у некоторой выдуманной планеты наличие небольших областей с ускорением свободного падения в несколько раз большим, чем на остальной поверхности планеты? Тип если представить, то допустим на планетке будет 1,5 м/с^2, а на трёх с половиной пятачках (по площади, относительно ко всей площади поверхности планеты) будет 10 м/с^2. Я вот думал, мол, если у планетки будет ядро вытянутое, как пельмень, а ещё если прямо аккуратно над складочками этого пельменя будет тонкая кора, то может быть и можно будет получить такие перепады. Есть ли ещё какие-нибудь варианты? Заранее спасибо, кст, чё там с вазимиром бля?
220 487146
>>7031
464565
221 487150
>>7145

>Возможно ли объяснить у некоторой выдуманной планеты наличие небольших областей с ускорением свободного падения в несколько раз большим, чем на остальной поверхности планеты?


Что такое небольших. Если у тебя планета крутится быстро, то на экваторе будет нормас, а на полюсах будет плющить. Есть классическая НФ про живущих в таких условиях.

Схема с пельменем работает только для очень маленьких тел, вроде изначального пельменя Пана. У них и ядра-то не может быть в классическом смысле. На таких телах и гравитация будет мизерной, пукнешь и улетишь нахуй. Есть ещё масконы в коре, но они во-первых не дадут настолько сильных вариаций, а во-вторых выравняются при сколько-нибудь сильной гравитации тела.

Так что нет, чем больше тело, тем больше его форма определена самогравитацией. Всё что имеет хотя бы 1g плюс-минус сапог, будет очень равномерным внутри. И чем больше самогравитация, тем более равномерным - нейтронные звезды например почти идеально шарообразные из-за этого. Единственный способ получить разную гравитацию на поверхности - раскрутить шарик. (но и у этого есть предел, твою планету распидорасит задолго до достижения невесомости на экваторе, ибо ты по сути контришь раскруткой самогравитацию, которая удерживает его от распидораса, начиная с некоторого значения)
222 487151
>>7145

>кст, чё там с вазимиром бля?


Нихуя, заглохло все
b9ee36165411a12542d4400x[1].jpg91 Кб, 400x400
223 487325
>>7150
НФ:
Хол Клемент - Экспедиция «Тяготение»
Вполне годный роман, очень легко читается, рекомендую
B2lforf2bi4.jpg118 Кб, 600x563
224 487329
А что, разве у SW1149 сферическое зеркало? Вроде в гугле пишут что параболическое. Где правда?
225 487333
Я думаю многие задумывались о том как может выглядить разумная форма жизни. Если так подумать, то получается, что они скорее всего тоже будут гуманоиды? Так же с двумя руками и ногами.
Я недавно навернул "Прибытие" и задался вопросом, а как инопланетяни построили корабль? У них там и стекло от защиты, и корабль весь такой ровный, а ведь пальцев и подходящих конечностей у них ведь нет. То есть понятно, что инопланетян хотят показать как можно более не похожих на нас, но что если это не верно с биологической точки зрения?
Разве эволюция может наделить разумом существо не могущее построить оружие, и вообще работать с мелкими и крупными деталями, а тем более прогрессировать до космических полётов?
Может всё таки как это не казалось банально, но инопланетяне (именно разумные и способные в технологический прогресс) будут всё таки гуманоиды и близко похожие на нас, как в фильмах Прометей и Аватар, а не кальмары или пауки бороздящие космос, что думаете?
sketch-1559551068193.png157 Кб, 720x1280
226 487334
>>7333
Я кстати только что задал этот вопрос в /b/, в треде про космос, намекая, что именно приматы в первую очередь всё таки идеальны для эволюции (как мы собственно видим). Но со мной начали спорить и говорить, что это не имеет значения, тупо рандом и что даже муравьи и слоны разумные.

Вы же тут люди посерьёзнее, поясните если я не прав. Может тут у вас и эволюционные биологи даже есть
227 487339
>>7333
>>7334
В сайфаче эта тема многократно обоссывалась ефремофагами. Основной посыл - да, для технологического и цивилизационного развития у инопланетян должны быть сложные рабочие конечности, поэтому то или иное конвергентное сходство с земными приматами неизбежно.
228 487342
Есть ли предел у расширения вселенной и каким он будет?
Я так понимаю расширение происходит благодаря растягиванию пространства/материи (я хз точно), но ведь это не может вечно быть. Может ли так быть, что она начнёт истончаться и рваться в итоге?
Может хуйню говорю, короче: как произойдёт смерть вселенной, теоретически?
image.png46 Кб, 200x200
229 487347
>>7342

>но ведь это не может вечно быть.


может

>Может хуйню говорю, короче: как произойдёт смерть вселенной, теоретически?


все планеты и звезды остынут. протоны развалятся и материя будет буквально рассыпаться. останутся черные дыры и фотоны которые летят в космосе бесконечно. потом черные дыры испарятся, фотоны потеряют энергию. не считая рандомных флуктуаций во вселенной не будет ни молекулы.

Абсолютно пустое пространство которое бесконечно расширяется. Понятие "время" потеряет смысл - им нечего будет мерять поскольку ничего не происходит и не меняется. И так будет бесконечно долго - никогда ничего больше не произойдет в абсолютно темной абсолютно пустой вселенной.
230 487349
>>7325
Ага, оно
231 487355
>>7333
Тащемта только в человеке есть божья искра.
232 487356
>>7347
Очень огорчает такая перспектива, хотя она и далёкая.
233 487360
>>7333
https://www.youtube.com/watch?v=niOk-tcN81I&t=2418s
Вот тебе часовая лекция на тему почему именно у нас появился разум, а у енотов\слонов\дельфинов нет.
235 487403
>>7347

>протоны развалятся


Це ж только гипотеза. Никто не наблюдал распад протона.
236 487449
>>7403
Распад ксенона тоже никто не ожидал

>Ученые зафиксировали одно из самых редких физических явлений — особый тип радиоактивного распада в ксеноне-124. Период полураспада ксенона, то есть время, за которое распадается половина вещества, достигает 1,8 умноженное на 10 в 22-й степени лет, то есть в триллион раз превышает возраст Вселенной.

15158439647690.png112 Кб, 500x500
237 487451
>>7403

>Никто не наблюдал распад протона


Ну это с какой стороны посмотреть.
238 487454
>>7449
Нихуясе. Это значит, что если мы будем просто хранить обычный ксенон, то у нас будут постоянно небольшие потери?
239 487455
>>4908 (OP)

>Что будет с человеком если он попробует ступить на поверхность газового гиганта?


У ГГ есть поверхность на которую можно ступить?
240 487456
>>7403
>>7451
РАСПАД ПРОТОНА СМОТРЕТЬ БЕЗ РЕГИСТРАЦИИ СМС
https://www.youtube.com/watch?v=Zs2qkpnFc2c
241 487484
>>7449
Но протон-то пока стабилен. Не засекли распад еще. Если он стабилен, то что тогда в итоге?
242 487489
>>7484

>Не засекли распад еще.


про ксенон до 2019 так же говорили и где он теперь?
243 487492
>>7489
Но про протон-то не говорят
244 487493
>>7492
двумя постами выше говорят
245 487494
>>7493
Причем тут твоя ракета?
246 487495
сложно
247 487496
>>7494
При том что я тебе ее за щеку сунул
248 487542
>>7454
Да, через септиллион лет у тебя останется половина!
249 487543
>>7542
Ну хоть что-то. На старость хватит.
250 487564
>>7455
Бамп
252 487599
>>7591
Что там, в крации?
sage 253 487600
>>7599
Очередной высер на ютюбчике.
254 487602
>>7599
Рассказывает почему именно у нас, как у вида, появился разум. У Дробышевского подача материала очень хорошая, легко слушается, ты начни и не заметишь как уже всю лекцию прослушал. Мы его компанией человек в 4-5 обычно смотрим, хорошо заходит.
255 487622
>>7564
Если какая-то твердая поверхность и есть, то только у каменного ядра в тысячах километров под видимой «поверхностью». Но инфа не 100%, внутрь газовых гигантов никто не заглядывал.

Во внешних слоях ничего твердого нет, там в основном только водород, плавно переходящий из газа в жидкий металлический.
256 487623
>>7622
Там вроде сверхкритика в металлический водород переходит.
257 487669
>>7602
А односложно?
rd-180flowdiagramcolor.jpg61 Кб, 1123x702
258 487670
>>4908 (OP)
Почему Охлаждение Кс ЖРД никогда не делают жидким кислородом, даже если это единственный криогенный компонент?
259 487675
>>7670
Потому что окислитель
260 487685
>>7622
Чому у юпитера столько разных цветов? Это всякие аммиаки, водяной пар, метан и тд?
261 487686
Алсо можно ли скажем зависнуть в атмосфере Юпитера на аэростате на такой высоте, где давление равно земному и при этом выйти наружу без вреда для здоровья? В подогреваемом герметичном костюме с подачей кислорода естественно.
262 487687
>>7686

> и при этом выйти наружу без вреда для здоровья? В подогреваемом герметичном костюме с подачей кислорода естественно.


Радиация быстро сделает в этом скафандре котлету по-киевски.
263 487688
>>7687
У юпитера мощная магнитосфера вроде как.
264 487691
>>7687
В радиационных поясах да. Но не в атмосфере, под ними.

>>7686
Нельзя, атмосфера водородная. Чем бы ты не надул шар - будешь тонуть, пока тебя не расплющит давлением. Любой летательный аппарат на Юпитере будет тяжелее воздуха. Летают там лишь аэрозоли и прочие дисперсные системы.

Кроме того, в атмосфере газовых гигантов дуют ебанутые ветра, на Нептуне вообще сверхзвуковые, на Юпитере попроще, но и их более чем хватит чтобы распидорасить твоего незадачливого испытателя в бесконечном шторме, вместе с шаром. Плюс на Юпе 2.5g - не просто жмёт сильно и тонешь быстрее, но и все аэродинамические процессы происходят быстрее.
265 487693
>>7455
Нет.

>>7622

>Если какая-то твердая поверхность и есть, то только у каменного ядра в тысячах километров под видимой «поверхностью».


Задолго до неё металлический водород, и он не жидкий, а твердый. Так что переход в любом случае плавный.
266 487694
>>7691
Можно вакуумом надуть сделать сверхпрочный дирижабль, который будет держать форму, даже если внутри давление меньше, чем снаружи.
267 487696
>>7694
нет
268 487697
>>7696
Что нет?
269 487698
>>7691
Ок, сделать вертолет для юпитера? А насчет ветра - так лететь просто вместе с ветром. Там же обширные зоны перемещающегося газа.
270 487699
>>7694
Теоретически можно, практически нельзя.
271 487700
>>7697
Нельзя.
272 487701
>>7698
Аппарат тяжелее воздуха - всегда пожалуйста. Но ему тоже пизда.

>А насчет ветра - так лететь просто вместе с ветром. Там же обширные зоны перемещающегося газа.


Даже эти обширные зоны в штиле никогда не бывают, дикая турбулентность там абсолютно везде.
273 487702
>>7699
Ну с современными технологиями наверняка. Но ведь по сути своей подводная лодка - это как раз такой дирижабль, только водоплавающий. Он, конечно, тяжелый что пиздец, но хуй знает, на что способны йоба-графены и прочая еще непридуманная муть.
274 487704
>>7670
Помимо уже упомянутой коррозии, у кислорода еще и так себе с теплоемкостью, у керосина и метана она более чем вдвое выше, у водорода — в 10+ раз.
275 487705
>>7691
Еще помимо дирижабля есть вариант с подогретым водородом. Собственно, многие воздушные шары так и летают.
276 487706
>>7693

>Задолго до неё металлический водород, и он не жидкий, а твердый


Именно что жидкий. Для твердого там горячо слишком.

https://science.nasa.gov/science-news/science-at-nasa/2011/09aug_juno3

>Jupiter's strangest feature, however, may be a 25,000 mile deep soup of exotic fluid sloshing around its interior. It's called liquid metallic hydrogen.

1520258711054.jpg34 Кб, 496x600
277 487708
>>7706

>Click on the image to launch a ScienceCast video about the mysteries of Jupiter's interior.


>ссылка ведет на jpg скрин видеоплеера с кнопкой Play


Ну еб их мать
278 487709
>>7706
Это на поверхности. Ниже следует фаза сверхтекучего твердого тела.
279 487710
>>7709
>>7706
>>7693
>>7622
там и не жидкость, и не твердое тело, а несколько экзотических состояний, не похожих ни на то, ни на другое при нормальных условиях
Кот.jpg11 Кб, 196x194
280 487713
>>7709
Чиво? Сверхтекучесть бывает только тогда, когда энергия частиц падает настолько, что квантовые эффекты начинают играть заметную роль, т.е. около абсолютного нуля, а внутри Юпитера много тысяч кельвинов. Ты явно перепутал со сверхкритической жидкостью (в которой от твердого тела нет ничего).

>>7710
Скорее там что-то между жидкостью и газом, упорядоченности твердого тела там явно нет.
281 487714
>>7713

>Сверхтекучесть бывает только тогда, когда энергия частиц падает настолько, что квантовые эффекты начинают играть заметную роль, т.е. около абсолютного нуля


Или при гигантском давлении. https://arxiv.org/abs/1702.00211

>упорядоченности твердого тела там явно нет


Сверхтекучее твердое тело (supersolid) это и есть упорядоченная решетка, проявляющая свойства сверхтекучести при этом. Вот такая экзотика, хотя казалось бы одно несовместимо с другим. При таких давлениях обычных соединений не бывает. спроси Оганова, он знает.
282 487715
>>7713

>Ты явно перепутал со сверхкритической жидкостью (в которой от твердого тела нет ничего).


В ней и от жидкости не так много как кажется. Но суперфлюиды сильно выше по фазовой шкале, чем металлический водород.
283 487717
У меня есть вопрос по астрологии.

НЕТ, УБЕРИТЕ РУКИ, Я ПО АДРЕСУ ЗАШЕЛ.

Так вот, способствовала ли астрология когда-нибудь развитию астрономии, например, открытием новых звезд или законов, или всегда сидела на шее и была абсолютно полностью бесполезным куском говна?
284 487718
>>7691
Ебать, Юпитер страшный.
285 487723
>>7717

>сидела на шее и была абсолютно полностью бесполезным куском говна


Думаю, что это. Зачем астрологам новые небесные тела? Им от таких открытий только проблемы одни--внезапно нужно придумывать как "учесть" "влияние" каких-то новых фигнюшек.
286 487730
>>7717
Опосредованно, позволяла легко выбивать деньги на изучение движения небесных тел и объяснять быдлу зачем тебе все эти дорогие железки.
2.png8 Кб, 467x236
287 487731
Если в центре звезд невесомость, то за счет чего проиходит синтез в ядре?
288 487733
>>7731
За счет давления, создаваемого вышележащими слоями. То, что вещество в самом-самом центре нихуя не весит и никакого вклада в обжатие ядра не вносит, никак не отменяет того, что все снаружи от него весит дохуя.
289 487780
Если за пределами вселенной нет времени и пространства, то получается, что человечество так и исчезнет ничего не узнав? То есть мы обречены ничего не познать?
Если пределы вселенной физически никогда покинуть не сможет, то как мы узнаем, есть ли например мультивселенная? Возможно всё, что нас окружает это лишь малая часть чего-то большего.

Что вообще наука говорит о том, что за пределами вселенной и можно ли нам посмотреть когда нибудь что там?
290 487788
>>7691

>сверхзвуковые


>ветра


Што
291 487790
>>7780

>То есть мы обречены ничего не познать?


Ща ебанут техническую сингулярность и все узнаем.
292 487791
>>7780
Ничего не говорит наука.
Никаких гипотез об этом строить нельзя, т.к. пока нет идей как что-то такое можно проверить.

Наука - не религия где ответы на все вопросы (и обычно в виде бога дыр), в науке можно сказать "я не знаю".

И ты можешь сейчас представить себе, что пока нет никаких оснований считать, что есть что-то за пределами вселенной, вся вселенная - это все что есть, больше ничего нет.
Я знаю, про пятно, но оно до сих пор ничего не доказало.
293 487792
>>7788
Ветер, скорость которого превышает скорость звука. Проблемс?
294 487806
>>7791
Гипотезы можно строить какие угодно. Другое дело, что пока не появится способа их проверки, ничего определенного об этом сказать нельзя. Тут ситуация хуже, чем с внутренностью черной дыры - если особо любопытный может упасть в дыру и проверить гипотезы о ней самостоятельно, то в отношении вселенной неизвестно даже способа ее покинуть.
295 487833
>>7792
Интересна физика процесса.
У нас тут на землящке посоны всякие хитрозаебущные форсунки и эжекторы да сопла изобретают, чтоб получить сз газовый поток, а там ветер дует прост потому что иди нахуй вот почему?
296 487834
>>7704
Это с учётом скрытой теплоты испарения?
297 487835
>>7788
Наиболее динамично развивающиеся ветры в Солнечной системе дуют на Нептуне — их скорость порядка 2400 км/ч
298 487876
>>7834
Нет, это просто теплоемкость. Кипения в охлаждающей рубашке вообще стараются не допускать, так как если образуется пар, то теплопередача через него сразу резко падает, и стенка может перегреться и прогореть.
299 487908
>>7833
Да просто большие БОЛЬШИЕ градиенты температур и давлений, вот и всё. Это же планета-гигант.
300 487930
>>7691

>Любой летательный аппарат на Юпитере будет тяжелее воздуха


пездеж. шар наполненный вакуумом
301 487931
>>7930

> шар наполненный вакуумом


Жидким надеюсь?
image.png12 Кб, 898x45
302 487933
>>7806

>Гипотезы можно строить какие угодно. Другое дело, что пока не появится способа их проверки, ничего определенного об этом сказать нельзя.


Нет, если твоя задумка непроверяема - это не научная гипотеза.
303 487934
>>7835

>км/ч


У нас спейсач или где?
Используй СИ.
304 487950
>>7933
Маня, способ проверки теориии - сюрприз! сюрприз! - диктуется самой теорией, и пока ты ее не построишь, ничего о ее проверяемости/непроверяемости ты сказать не сможешь.

А вообще, есть эмпирическое правило - если кто-то в дискуссии на бордах первым вспоминает про критерий Жоппера, то он с вероятностью, близкой к 100%, есть чухан, говноед, стукач и провокатор.
Стикер512x512
305 487951
>>7950

>если кто-то в дискуссии на бордах первым вспоминает про критерий Жоппера, то он с вероятностью, близкой к 100%, есть чухан, говноед, стукач и провокатор.


Заметьте, я про него ничего не говорил, я лишь формальное определение гипотезы дал, а вот ты его первый упомянул.
306 487952
>>7950
И да, какие нахуй теории, если речь шла о гипотезах? Ты даже эти термины путаешь?
Неудивительно, что ты сам под свои определения и попадаешь.
307 487960
>>7951
>>7952

>я про него ничего не говорил ррряяяяяя!!!1


Оно у тебя на приклейтеде, чухоня. И не бугурти ты так, аж два поста высрал ))))

>какие нахуй теории, если речь шла о гипотезах


Я говорю в терминах теоретической физики (где любой пук+вскукарек считается теорией), а не в терминах википедальной школоты.
image.png55 Кб, 225x225
308 487961
>>7960
Всё с тобой понятно. Можешь продолжать, это забавно.
1559362867195.jpg342 Кб, 1510x1148
309 487963

>M87zoom.webm


Как так зумируют вообще?
А можно ли сделать такой же, но чтобы смотреть микробов на жопе человека на другом континенте?
310 487964
>>7963
Нет.
311 487965
>>7963
Там не зум с одной камеры, а разные камеры, разные зумы, спектры.
312 487975
>>7950

>чухан, говноед, стукач


А вот и биомусор из /b/ подтянулся.
badumtss.png413 Кб, 783x776
313 487978
>>7961
>>7975
Подрывом википедальных школотунов удовлетворен:3
314 487979
двачую
315 488023
Только сегодня узнал реальную причину невесомости на МКС. Тогда сразу тупой вопрос: насколько невесомость дальше от земли будет отличаться от невесомости на станции? Скажем, где-нибудь между Марсом и Землей. Без всякого ускорения. Просто прилетел туда и вышел из корабля в открытое странствие.

Если точно такие же свойства передвижения будут, то почему?
316 488026
>>8023
потому что там будет орбита вокруг солнца
317 488027
>>8026
Параметры гравитации, свободного падения и прочего не отличаются нигде в космосе?
318 488029
>>8023
>>8027
For all intents and purposes, ты не почувствуешь разницы находясь на орбите, в свободном падении (что одно и то же) или в пространстве вдали от всех источников гравитации.
https://ru.wikipedia.org/wiki/Принцип_эквивалентности_сил_гравитации_и_инерции
319 488030
>>8029
Кстати, почему я один только вставляю ссылки на русском языке без %АУЕ%ХУЙ%228 и т.п.?
Знаю, что нерилейтед, но почему до сих пор ссылки корежатся у людей?
320 488031
Я не понел, почему если запустить с орбиты земли аппарат на очень вытянутую орбиту (скажем чтобы она пересекала орбиту юпитера), то аппарат все равно вернется к земле если не окажется близко к юпитеру или типа того? Ведь гравитационное поле земли на таком расстоянии вообще неочем и почти не влияет на космический аппарат, даже солнце влияет больше. Почему он не полетит просто дальше по параболической траектории и не станет спутником солнца, а непременно вернется в периапсис земли?
321 488033
>>8029
Спасибо.
322 488034
>>8030
Не ты один. Вообще эти проценты это стандарт https://en.wikipedia.org/wiki/Percent-encoding и 99% браузеров кодируют урлы в такую еботу по хорошей причине. Просто не все знают что нужно поставить пробел в начале, чтобы отказаться от кодировки.
323 488037
>>8031
Если будет пересекать орбиту Юпитера - не вернется. Орбита должна быть внутри зоны гравитационного влияния (т.е. влияние тела должно пересиливать иные возмущения), иначе не вернешься.
324 488038
>>7963
Можно, снимаешь последовательно кучу картинок всё ближе к жопе, а финальные кадры вообще микроскопом. Склеиваешь это все в один видеоряд и получаешь микробов с другого континента. https://www.youtube.com/watch?v=INqKqk-ifpc
325 488039
>>8037
Что значит должно пересиливать? А если влияние других тел такое же незначительное, как влияние земли? Вообще странно, симуляторы всегда показывают, что как бы вытянута орбита ни была в пределах возможностей симулятора, периапсис всегда находится в точке старта. Видимо не учитывается влияние других тел
326 488042
>>8039
значит что тело начнет притягивать аппарат настолько сильно, что он изменит орбиту или вообще останется на орбите нового тела

>симуляторы всегда показывают, что как бы вытянута орбита ни была в пределах возможностей симулятора, периапсис всегда находится в точке старта


потому что надо именно неподалеку пролететь, чтобы было влияние. Например даже в КСП орбита меняется после близкого пролета иного тела
5-Figure4-1.png44 Кб, 928x464
327 488043
>>8039
То и значит. На определенном расстоянии от тела влияние какого-нибудь другого тела уже будет сильней. Причем это расстояние не всегда одинаково, вот пролетит планета поближе и захватит спутник на высокоэллиптической орбите другой планеты, который как раз к ней поближе подошел. На пертурбациях низкоскоростных участков орбиты основана куча хитровыебанных низкоэнергетических траекторий, например баллистический захват, позволяющий магически превратить высокоэллиптическую орбиту вокруг одного тела в круговую вокруг другого.

Вокруг любого тела есть сфера Хилла, она же сфера гравитационного влияния, которая на самом деле не сфера, а полость немного неправильной формы с постоянно изменяющимися границами, т.к. соседние тела "теснят" её и движутся относительно неё. Внутри сферы Хилла доминирующей силой является гравитация тела, и спутник всегда движется по замкнутой орбите, если не превышает вторую космическую скорость. (в свою очередь падающую с дистанцией до тела). На границе сферы Хилла вторая космическая скорость обращается в 0 (как в точке Лагранжа), пукнешь и улетишь.

>Вообще странно, симуляторы всегда показывают, что как бы вытянута орбита ни была в пределах возможностей симулятора, периапсис всегда находится в точке старта


Это какие? То что ты описываешь, выглядит будто симулятор тебе рисует оскулирующую (кеплерову) орбиту, или вообще пиздит. В орбитере, например, он во многих случаях не учитывает вторую космическую. В любом случае неважно, это все предсказание и способ рисования в конкретной программе.

Рекомендую поломать себе мозг об GMAT https://sourceforge.net/projects/gmat/ , выполни тамошние туториалы хотя бы, и очень многие вопросы по орбитальной механике отпадут. По эллипсам аппараты летают только в учебниках.
328 488046
>>8031

>Я не понел, почему если запустить с орбиты земли аппарат на очень вытянутую орбиту (скажем чтобы она пересекала орбиту юпитера), то аппарат все равно вернется к земле если не окажется близко к юпитеру или типа того? Ведь гравитационное поле земли на таком расстоянии вообще неочем и почти не влияет на космический аппарат, даже солнце влияет больше. Почему он не полетит просто дальше по параболической траектории и не станет спутником солнца, а непременно вернется в периапсис земли?


Так ведь не вернется. Тебе надо очень тонко расчитать чтобы возвратная солнечная орбита пересекала земную притом в тот момент когда там находится земля.
В космосе тебе надо думать в четырех координатах, еще и время учитывать. Кербач в этом плане отлично помогает, как и ранние видео Скотта "Наше Всё" Мэнли.

>>8034
Я без пробела. В стандартном хромоге надо пробел, а в лучшей опере (которая Вивальди) - такого нет, сразу искаропки делает.
329 488047
>>8046

>а в лучшей опере (которая Вивальди) - такого нет, сразу искаропки делает


Она искаропки нарушает стандарты, они не зря придуманы. Но хуй с тобой, здесь не /s/ все же.
330 488050
>>8043

>баллистический захват, позволяющий магически превратить высокоэллиптическую орбиту вокруг одного тела в круговую вокруг другого.


А насколько долго такой захват может продолжаться? Есть же какой-то механизм который и позволил захватить спутники в круговые орбиты, а не вышвыривать их на пятом проходе.
Дай угадаю: время? Захвачены были миллионы, уселись единицы, удачно попав в синхрон, это так?

>>8047
У меня не один бровзер, я выбрал для серфа тот, который как раз очень удобно делает все вообще.
Не та самая Опера, что раньше, но ближе всего к ней.
Сейчас гугель намудрыжит с движком и окончательно к лисам перееду, че тут думать. Это не религия для меня, как для некоторых, есть что лучше - буду юзать.
331 488058
>>8050

>А насколько долго такой захват может продолжаться?


Ну мы же об искусственных спутниках-то говорим вроде, поэтому быстро, если правильно рассчитать. Dawn при подходе к Церере если не ошибаюсь использовала баллистический захват, управилась быстро. Да и многие другие спутники так летали. Но да, конечно, любая низкоэнергетическая траектория это всегда размен дельты на время. Все интересные процессы в таких траекториях происходят на очень медленных участках траектории, например в случае баллистического захвата нужно чтобы и без того малая пертурбация оказалась больше чем текущая орбитальная скорость.

Вообще, у людей незнакомых с орбитальной механикой низкоэнергетические траектории ломают шаблон обычно потому, что они плохо представляют как спутник движется в разных точках отсчета. Это реально не интуитивно и способно по первости взорвать мозг, потому я и посоветовал GMAT, это первое чему там учат в туториале, очень наглядно.

>Захвачены были миллионы, уселись единицы, удачно попав в синхрон, это так?


Ну в случае естественных спутников так и есть, например Феба скорее всего так и была захвачена Сатурном (а затем частично эродировала в кольцо), уж очень необычная траектория, как раз во внешней части SOI Сатурна.
Без названия.jpg302 Кб, 2743x2400
332 488066
Почему просто нельзя тупо поставить линзу(или несколько) из полимеров на L1 пару квадратных км3 и ещё приемники-ретрансляторы на геостационарке, что бы пучек бил в огромную. черную железную бочку с солью и водяным контуром 24/7

Можно ведь запросто 50-100 тераватт поднять

щит 1000км2 на Л1 может убрать 1% излучения(тыщи террават в сутки)

этож блядь гениально

Возможно полотна из полимера 100 на 100 метров хватит на 10 Россий

а че будет с атмосферой если такой лазир её будет прорезать круглый год?
333 488067
>>8066
пучекю
334 488068
>>8067

ЛАЗИР
335 488069
>>8068
А прикинь, лазир такой пучеками света - БДЫЩ, БДЫЩ! этож блядь гениально
14252463463.jpg28 Кб, 512x422
336 488070
>>8069
Но ведь л1 орбита стабильна(с двигателями) и гораздо ближе к солнцу, можно реально прислать на землю несколько сот тыщ магаватт по цене 2-3 атомных станций

Или атмосфера сломается?
337 488071
>>8050

>А насколько долго такой захват может продолжаться? Есть же какой-то механизм который и позволил захватить спутники в круговые орбиты, а не вышвыривать их на пятом проходе.


А, только сейчас понял суть вопроса. При идеальном совпадении сразу же и захватывается, без каких-либо дополнительных танцев. При неидеальном может и телепаться куда-нибудь. Но идеальное, как ни странно, более вероятно.
338 488072
>>8070

Прислать в квадрат 100 на 100 метров*
339 488074
>>8070

>гораздо ближе к солнцу


Ага, аж примерно на процент ближе Земли. Поэтому мощность излучения Солнца там будет в целых ~1,02 раза выше чем на Земле. Ах ну да, круглосуточно и без влияния атмосферы. Ну пусть со всеми поправками в 3 раза больше. Чтобы получить этот гигантский выигрыш, ты должен 1) доставить и развернуть эту некислую ебалу в L1; 2) ворочать ее так, чтобы фокусировать пучеки в нужную точку на вращающейся Земле; 3) постоянно корректировать орбиту движками, ведь L1 нестабильна. Что-то мне кажется, что оно того не стоит, проще засадить 3 кв. км. Земли солнечными батареями, чем ебаться с 1 кв. км паруса в 1,5 млн км. от Земли.
340 488076
>>8074

1000km2 тени на л1 блокирует 1% света на землю(тыщи террават в сутки(мы потребряем 70 в год)

Бить будем в отражатель на удобной геостациорарочке, а он уже будет работать по бочке, можно вообще импульсно, отражатели будут налетать на тераватный сгусток энергии раз в час нагревая ебический буфер размером со стадион до тыщ градусов

Это в сотни раз дешевле любой штуки на земле
341 488077
>>8076

>1000km2 тени на л1 блокирует 1% света


Далеко не 1%. Учитывая близость L1, можем пренебречь углом. Земля имеет радиус 6350 км, что дает площадь круга в 126 млн кв. км. Отсюда 1000 кв. км. это менее 0,001%.

Прикинь к тому же стоимость запуска этой хрени (100 кв. км будут весить ДОХУЯ, из чего бы ты их не сделал). Представь технические сложности создания линзы, ОЧЕНЬ точно держащей форму при таких размерах, ведь малейшее искажение - и свет пойдет не в приемник а сожжет Воронеж, или, в лучшем случае, улетит к Нептуну. Прикинь затраты топлива на корректировку орбиты этой дуры, ведь L1 нестабильна. Учти необходимость замены через полвека - ведь метеориты (при таких размерах столкновения неизбежны) и пыль будут дырявить, царапать и замутнять твою линзу.

>можно вообще импульсно


Ага, ну то есть ты еще вдовесок собрался ставить на свою йоба-линзу солнечные батареи, аккумулятор и излучатель.
342 488109
>>8074
Самая мякотка что это буквальный луч смерти в случае (не)сознательной потери фокусировки на приемнике.

Я за орбитальные солярки, тема неплохая, но у нас на планете вполне есть варианты качественно и не сря добывать энергию: ТОРИЙ (И уран).
343 488111
>>8074

>Ага, аж примерно на процент ближе Земли



Давайте тогда пояс из линз или зеркал максимально близко к солнцу на солнечную орбиту!

Луч будет импульсно бить по системе отражателей на орбите земли а они уже фокусировать на бочку с солью.
344 488114
Какова природа такой необычности у звезды Пшибыльского?
ketmitthinking.png2 Мб, 1860x2190
345 488131
Как работает радиация? Посмотрел сериал чернобыль и не понял.
Графит же вроде сам по себе не радиоактивный?
А что там радиоактивное было?
И вообще почему материя накапливает ридиацию, а не отталкивает, если радиация это просто лучи.
Я не понял.
346 488132
>>8066

>щит 1000км2


че блять? знаешь сколько будет это весить и сколько стоить вывод? к тому же эта хуйня за пару лет продырявится солнечным ветром и микрометеоритами.
347 488133
>>8131
Потому что изза ядерной реакции вылетают нейтроны. Они летят, врезаются в другие атомы (графита например) и прилипают к ядрам. От чего те становятся нестабильны и разваливаются постепенно, излучая нейтроны, гамма-излучение, бета-излучение и прочую парашу. Ты сам будешь излучать, если на тебя посветить нейтронами.
348 488134
>>8133
Нейтроны это же электричество, не?
yobapalm.jpg10 Кб, 229x220
349 488135
>>8134
троллинг тупостью?
350 488136
>>8135
Ну я давно вроде читал что нейтроны бегают по проводам или что-то такое. Поясни пожалуйста.
your brain.png218 Кб, 640x358
351 488140
>>8136
Тебе б читнуть чего-нибудь. Хотя бы шапку у статьи на википедии
https://ru.wikipedia.org/wiki/Электрический_ток
352 488141
>>8140
Перепутал нейтроны с электронами, бывает.
А ведь материя состоит из атомов, В которых есть нейтроны и протоны - почему всё не радиоактивное?
353 488146
>>8131
Тебя конкретно поражающие факторы интересуют?

Бывает гамма-излучение, которым светятся радиоактивные материалы. Это тот же самый свет, только круче. Круче ультрафиолета, круче рентгена, круче неба и аллаха. Очень хорошо проникает во всё и ебёт живые клетки и ДНК. От неё только одна защита - толстый слой плотной хуйни, или отойти подальше от фонящего материала. Графит от реактора сам излучал гамму, ибо был облучен прямо в реакторе сильным потоком нейтронов. Это наведенная радиоактивность.

Бывают еще частички радиоактивного материала. Они проникают в виде пыли, накапливаются в грибах, растениях, животных, земле. Проглотишь таких дохуя, сожрешь гриб такой, и они будут облучать тебя изнутри. От пыли соответственно защита противопылевая (очки, респиратор и т.п.), ну и химическая иногда. В воздух была выброшена именно такая хуйня, и район накрыло именно оно, рыжий лес оттуда же стал рыжим.
354 488147
>>8146
А почему радиация так слабо повлияла на растения?
355 488149
>>8146

>рыжий лес оттуда же стал рыжим.


Чо это он рыжий?
356 488162
>>8149
Потому что стал рыжим. Читать не умеешь?
357 488164
>>8162
Охуенно объяснил, почему масло масляное? Потому что стало масляным.
358 488165
Дано: есть обитаемая планета с жидкой водой. Наибольшая часть ландшафта - пустыни с оазисами, оазисы небольшие, но многочисленные и относительно близко к друг другу расположенные. И это именно пустыни, с песчаными дюнами и всё такое. Так вот при каких условиях - размер планеты, её состав, тип родительской звезды, положение планеты вокруг этой звезды, тип орбиты, скорость вращения, угол наклона оси, наклон прецессии, наличие спутника и его тип - на этой планете ветрищ не будет и дюны не будут надувать ветров в оазисы? Я имею в виду не полное отсутствие ветра, а просто его слабость.
359 488166
>>8165
Ну, по идее, чем однороднее климат, тем слабее ветер. Отсюда я бы сказал, минимальный наклон оси, почти круговая орбита, равномерный рельеф без гор и океанов. Быстрое вращение, чтобы цикл день/ночь не давал больших перепадов температуры.
360 488167
>>8166
Еще, вероятно, плотная и толстая атмосфера поможет.
361 488169
>>8165
А ничего что дюны это следствие ветров?
362 488171
>>8169
Допустим, в ранние геологически годы было буйство ветров, но вот сейчас такого нет.
363 488172
>>8171
Так они рассосутся со временем, че ты несешь вообще
Сделай тогда оазисы на холмах только хуй знает как там вода будет тогда
1523429334783.jpg728 Кб, 900x900
364 488183
А вот почему нельзя в невесомости долго находиться? Из-за вреда для костей только? Нельзя ли как-то что-то сделать с самими костями, создать таблетки, которые замедлять вымывание кальция из организма, вместо того чтобы создавать огромные структуры из вращающихся центрифуг? Если взвесить тяжесть изобретения лекарственного решения проблемы и тяжесть изобретения и конструкции центрифуги, то лекарства и таблетки гораздо легче создать, так ведь?
365 488184
>>8183

>гораздо легче создать


Это не программирование, где достаточно написать

void tabletka () { this->calcium_vymivanie_rate /= 10; }
366 488185
>>8184
Да это может быть не просто, но полезней и для многих других заболеваний костей, может иметь пользу на Земле даже. И всё равно это может оказаться проще, чем создавать огромные центрифуги для дальних полётов в космос. Знаю, что там еще мышечная деградация, но её вроде как упражнениями останавливают. Медицинское решение может быть также комплексным. Это могут быть таблетки таргетирующие печень, в дополнение к каким-нибудь экзотическим мерам вроде обогащения костей редкими изотопами кальция, которым сложнее принимать участие в реакциях организма.
367 488186
>>8185

>редкими изотопами кальция, которым сложнее принимать участие в реакциях организма


Изотопы достаточно тяжелых элементов практически не отличаются друг от друга по химическим свойствам. Даже у водорода (легче уже некуда) отличия между протием и дейтерием не такие уж большие.
368 488189
>>8186
ну, может быть. Я еще где-то читал, что подмена изотопов в теле может сделать человека более устойчивым к радиации. Типа кальций, углерод, воду - всё заменить более тяжелыми стабильным изотопами и тогда излучение из космоса снизит урон, так как не сможет разбивать атомы так легко, повреждая днк и другие мелкие структуры.
369 488192
>>8189
Тут не знаю, но вангую, что это хуйня. Насколько я понимаю, самыми уязвимыми для радиации будут легкие атомы, то есть водород. А вот его заменять, по крайней мере в значительных количествах, не стоит. Пушо как раз дейтерий слабоядовит. Может, процентов 10 и можно заменить, но вряд ли больше, иначе бо-бо. Так или иначе, наверняка там речь идет не об охуенной защите, а "можно сделать человека на 3% устойчивее к радиации".
370 488194
У меня вопрос к оп-посту.
Что значит вопрос:

Q: Можно быстрее?
A: Можно упасть в пузырь альбукерке, наса уже почти надула его.

Немного упорот, извините.

Заранее спасибо!
371 488195
>>8194
Там пузырь Алькубьерре подразумевается.

Вкрации: по современным пердставлениям, двигаться в пространстве быстрее скорости света в вакууме нельзя. Но нет ограничений (или, скорее, они просто неизвестны) на то, как может корежиться пространство. Идея в том, чтобы сделать "следующую точку" твоего маршрута ближе, нужным образом "подкорректировав" пространство. То есть ты его "перед собой" скукоживаешь, продвигаешься по этой скукоженной части, а потом раскукоживаешь ее обратно до оригинальных размеров. Такой себе варп-движок в научной формулировке.
372 488196
>>8195
Спасибо.
373 488197
>>8195
Так а шо там у насы?
374 488198
>>8192

>дейтерий слабоядовит


да, в больших концентрациях. Я давно читал статью, но там речь идет о продолжительном обогащении организма сверхмалыми дозами, пока он не начнёт заменять значительный процент в организме.
375 488199
>>8198
Так дейтерий как раз ядовит если его в организме дохуя, без разницы, накопилось так или магически сразу все атомы заменились. Ты можешь без особых последствий хоть пол-литра тяжелой воды ебануть, главное чтобы однократно. А вот если так каждый день делать, то начнутся проблемы. А если каждый день пить по наперстку (и 2-3 литра обычной воды, как всегда), то он и накапливаться не будет. Ты его выссышь точно так же, как обычную воду.
376 488201
>>8197
Не слежу за новостями, но вангую, что ровно нихуя. Это из разряда "упражнений для ума", судя по всему.
377 488211
>>8199
Ок, я тебе тогда ссылку на статью кину, может я не так перевел её что-ли
http://www.oncotarget.com/index.php?journal=oncotarget&page=article&op=view&path[]=24461&path[]=76813

В параграфе Deuterated organic compounds говорится про токсичность, но также замечается что это имеет очень сильный анти-раковый эффект и может увеличивать долголетие в определенных случаях. Может дело в том как это балансировать, можно в периоды риска вспышек подменять воду на тяжелую воду в рационе питания космонавтов.
378 488217
>>8211

>Conclusions


>...While many of the strategies proposed above may seem speculative, they should be considered as a foundation for future research directions.


Короче, по сути ничего практически полезного (с нынешними технологиями) тут нет.

Хотя там есть то, о чем я не подумал: радиация приводит не только к непосредственным повреждениям в той же ДНК, но и может вызывать ионизацию всяких кислородов, которые уже потом чисто химически ебут все вокруг. И дейтерий более полезен как защита от этого, потому что связи D-C крепче, чем H-C (с протием, то бишь).

Но один хуй, непонятно как ввести в организм достаточно дейтерия чтобы его с одной стороны не убить кхуям, и с другой стороны так, чтобы в ДНК его стало МНОГО. ДНК ведь генерируются не централизованно, а при размножении клеток, то есть по всей тушке. А если ты не можешь разделить дейтерий "по назначениям", то в ДНК его будет столько же, сколько во всем организме в целом, плюс-минус. То есть максимум процентов 10-20 от всех атомов водорода, иначе пизда котенку. Короче, я все равно не вижу, как это может качественно улучшить радиозащиту, разве что количественно и не в разы.
379 488218
>>8217
Ну так там и про углерод-13 говорится. Ты видать прост проф деформировался и думаешь, что чтение конлюжн является заменой чтению статьи. В то время как в статье идут широкие намеки на то, что практически полезного навалом, но нужно продолжать исследования. И эти исследования могут сэкономить метры защиты от радиации. Гораздо удобнее защитить человека от радиации изнутри, также как и защитить от деградации костной ткани изнутри, а не при помощи центрифуг ебаных.
380 488241
>>8146

>аведенная радиоактивность.


И какие там были изотопы. Насколько я знаю, наведенной радиоактивности подвержены далеко не все материалы и графит с водой в реакторах и используют потому что они слабо "набирают" радиацию. Графит, мне думается, от контакта с радиоактивными материалами со временем просто ими пропитывается, радиоактивными изотопами стронция, цезия, йода и хуй знает чем еще, в продуктах деления урана чего только нет.
381 488244
>>8147
Растения слабо подвержены действию радиации, в среднем смертельная доза для растений в несколько тысяч раз выше чем для человека.
pepe.png106 Кб, 740x982
382 488271
На какую максимально орбиту солнца можно пихнуть зеркало/линзу и сколько там будет ватт а метр?

Роскосый сможет в недосферу дайсона? можно ли концентрированное световое излучение в тыщи терраватт с помощью приемника на L1 переконвертировать с потерями в какой нибудь микроволновое излучение что бы безопасно передавать на землю?
383 488273
Нахуя? Тут термояд на подходе.
384 488274
>>8201
Я чет слышал вроде, что какие-то небольшие успехи у них есть.
385 488275
>>8273

Нету денег на столько термояда, ИТЭР стоит 15 лярдов за 1000 мвт тепловой энергии. Несколько сот зеркал 100 на 100 метров выйдут на круг в 100 раз дешевле.

Для того, что бы перевооружить только текущую экономику, нужно построить кабанчиком более 5000 АЭС, это 60 триллионов долларов, А термояд стоит буквально дороже в 10 раз. Недосфера дайсона будет стоить максимум с десяток МКС.

Даже на солнечные панельки перевооружить нету ресурсов, серебру придет крышка в 2030.
386 488279
>>8271
Дохуя таких проектов было, все сыпались именно на беспроводной передаче энергии.
387 488280
>>8141
Потому что сильное взаимодействие.
388 488286
>>8275

Можно просто чуть-чуть пустыни застроить солнечными батареями/тепловыми генераторами. Никаких сложных космических технологий.

> Даже на солнечные панельки перевооружить нету ресурсов, серебру придет крышка в 2030.



О, это штото новенькое. Раньше /зоговцы и пидорашки обычно пугали заканчиванием редкоземельных элементов.
389 488290
>>8286

Нету столько серебра для 70 террават панелей
390 488307
>>8290

>Нету столько серебра для 70 террават панелей



а) Ты спутал "разведанные запасы серебра которые рентабельно добывать по текущей цене" c "нету".
б) есть заменители. менее эффективные но и технологии не стоят на месте.

Ну и если действительно будет все плохо солнечные паровые турбины/машины /двигатели стирлинга нам в помощь.
391 488323
>>8275

>нужно построить кабанчиком более 5000 АЭС


Экономисты уровня /b.
392 488335
>>8323

Ты знаешь энергобаланс угля, нефти и газа? он колоссален, мы говорим не только про электричество, но и про транспорт, промышленность, строительство, дофига всего.

+ что бы заметно поднять уровень жизни на планете нужно в разы больше добычи и производства даже с учетом растущей энергетической эффективности.

Единственный доступный по цене путь, это отправить на близкую солнечную орбиту группу зеркал и линз, что бы прислать сюда концентрированный солнечный зайчик в тысячи тераватт, а строить тысячи термоядерных электростанций которые в 10 раз дороже атомных это чистое безумие, я не понимаю, зачем вообще на это даже деньги тратят, цена термояда всегда будет запредельно космической, достаточно пролистать количество и качество оборудования на сайте ИТЭР, это чистое безумие.

Так же с ВИЭ, что бы закрыть текущий энергобаланс нужно наштопать панелей размером с 70% Испании а промышленность должна быть уже сейчас в 37 раз больше чем сегодня, что бы выдать это хотя бы к концу века, и стоить это все будет просто астрономические деньги.
393 488340
да
image.png637 Кб, 600x600
394 488342
>>8335

>ИТЭР


>Экспериментальный


Щас бы по экспериментальным прототипам судить о серийных.
395 488343
>>8273
>>8335

>стоить это все будет просто астрономические деньги


Ага, ну а "группу зеркал" ты тип почти бесплатно отправишь, ну да.

Каждое надо будет снабдить двигателем, ведь они будут быстро вращаться вокруг Солнца, значит угол с Землей будет постоянно меняться. Поскольку они будут близко к Солнцу, температуры там будут адские. Несмотря на это, материал зеркал не должен подвергаться температурной деформации (иначе нифига не на приемник они отражать будут). И двигатели тоже должны работать бесперебойно.

Приемник должен не расплавиться от гигантского количества проходящей энергии. То есть, либо он должен быть коллосальных размеров, либо их должно быть много. Ну или ты собрался светить прямо на Землю, со всеми вытекающими последствиями для экологии, а также с лучами смерти при любом сбое "прицеливания", выжигающими города.
396 488345
>>8342

Даже если он выйдет на цену ВВЭР-1200 магически за меньше чем 10 лет, то нужно будет строить тысячами, если не десятками тыщ.

Пару сотен запусков сверхтяжей с полимерными линзами и зеркалами как то дешевле будет на порядок.
397 488346
>>8345
Миллиардами, чо уж.
398 488347
>>8343

>Каждое надо будет снабдить двигателем, ведь они будут быстро вращаться вокруг Солнца, значит угол с Землей будет постоянно меняться.



Зачем, сами зеркала будут неподвижными, просто будет импульсная система, зайчик будет налетать на приемник каждый час или даже каждые 10 минут(скорость вращения близко к солнцу очень высокая) это будут просто тупые отражающие куски, естественно никакие рули и автоматика не выдержит и год на тех орбитах.

Вот запил приемника(ков), это да, но в целом если такой массив энергии можно будет переработать в безопасные волны где нибудь на L1 и спускать дальше на землю, даже с огромными потерями, то будет огромная экономия по ёмкости материалов и цене если сравнивать с термоядом которого ещё не будет лет 5(если опять не отодвинут) или панельками и батареями.

Что, бы солнечные панели заменили уголь и газ хотя бы в электричестве на 60% к 2050 году, скорость их развертывания с 2015 года должна была быть в 6 раз быстрее чем сегодня.
399 488348
>>8346

Тебе сложно посмотреть на википедию где написано число реакторов, какой процент они занимают в генерации электроэнергии и какой процент сама электроэнергия занимает в энергобалансе земли?

ты идиот?
400 488350
>>8347

>(скорость вращения близко к солнцу очень высокая)


Все ясно, ты - неадекват. (если хочешь размещать зеркала на низкой солнечной орбите на существующих технологиях и знаниях)

> это будут просто тупые отражающие куски


Это невозможно. Без систем ориентации там все быстро собьется - в том числе от солнечного ветра, светового давления, неравномерностепй гравитации и так далее.

> Что, бы солнечные панели заменили уголь и газ хотя бы в электричестве на 60% к 2050 году, скорость их развертывания с 2015 года должна была быть в 6 раз быстрее чем сегодня.



Ты забываешь ветряки которые являются сейчас основной альтернативкой (сюрприз)

Но это по сути не важно - гораздо проще нарастить производственные мощности известной технологии в 6 раз чем запилить систему к которой 95% технологий просто нет в принципе и неизвестно когда будет. Например что бы чтото хотя бы на орбиту меркурия пульнуть нужно столько дохуя дельты что там для мелкозондлов нужен либо свертяж либо много гравиманевров.
401 488353
>>8350

>Все ясно, ты - неадекват. (если хочешь размещать зеркала на низкой солнечной орбите на существующих технологиях и знаниях)



Это возможно, и гравитацию с ветром можно просчитать, даже в целом и автоматика возможна, просто защита нужна будет тяжелая, но зеркало, даже очень маленькое на очень нзкой солнечной орбите сможет собрать сотни или даже тыщи тераватт,

Цена на ракету не проблема, двигатель Raptor готовят к 20 долларам за килоньютон например(50 ре-юзов) вполне себе копеешно получается если так прикинуть.
402 488354
>>8348
Просто иди нахуй, прожектер.
403 488355
>>8354

>маньке припекает от факта, что экономика существует.



Ясно.
404 488356
>>8347

>импульсная система


Хуимпульсная. Нахуя вообще твоя ебала тогда нужна, если она работает одну секунду в три дня? Или ты собрался аккумулятор еще вкрячить в свое зеркало?

Я, кстати, вообще не уверен, что возможно, по крайней мере с нынешними технологиями, создать зеркало, хотя бы статическое, которое светит ровно в заданную точку за сотню миллионов километров.
405 488357
>>8355
Даун настаивает на своем утопическом прожекте, хотя ему уже стопицот раз пояснили, почему это невозможно в ближайшие сто лет как минимум. Пиздуй нахуй короче, школьник.
406 488362
>>8356

>Хуимпульсная. Нахуя вообще твоя ебала тогда нужна, если она работает одну секунду в три дня? Или ты собрался аккумулятор еще вкрячить в свое зеркало?



Так их там будет много, несколько тыщ на одной и той же орбите, буфером будут бочки с солью на земле или ещё какая нибудь ебала размером со стадионы, несколько тыщ таких по всему шарику.
1486552740421.jpg83 Кб, 1080x1080
407 488368
Хочу написать сцай-фай рассказ, но нужно чтобы было как-то минимум твёрдо и чтобы там были межзвездные путешествия. Огромный корабль с двигателям на металлическом водороде, при пролёте мимо звезды назначения делает маневр у планеты гиганта и сбрасывает пассажирский модуль, который тормозят лазерами с орбиты луны газового-гиганта. Сам межзвездный корабль продолжает путешествие к другой звезде без торможения.

Есть ли дыры в такой схеме, на которые потом будут плевать, как на нетвердоту?
408 488369
>>8368

>делает маневр у планеты гиганта


>продолжает путешествие к другой звезде без торможения


Если он не тормозит с межзвездных скоростей, то любой маневр будет пуком, он, считай, ни на что не повлияет.

Ну и плюс на таких скоростях любой мимокамушек (которых в звездных системах полно) станет мегабомбой. Хотя тут можно сослаться на неведомую йоба-защиту. В конце концов, за пределами звездных систем тоже можно об камушек споткнуться, так что какая-то защита один хуй желательна.
409 488370
Если земляне построят звезду смерти, которую будет можно наблюдать с земли не намного хуже луны, она будет влиять на приливы-отливы, лунатиков, оборотней итд.?
410 488371
>>8369
Ок, напишу, что лазером на пути следования корабля всё сжигают нахуй, чтобы ни одного камешка. Маневр у гиганта нужен лишь чтобы лазер не был каким-то уж совсем йоба стреляющим на полсистемы. На луне-гиганта стоит много электростанций, которые питают лазер, убивают скорость капсулы с экипажем. Капсулу подбирает корабль-буксир и тащит куда нужно.
411 488372
>>8370

>она будет влиять на приливы-отливы


Да. Примерный перепад гравитации от тела на противоположных точках Земли равен G m (1/(R-r)^2 - 1/(R+r)^2) = G m 4 Rr / (R-r)^2 (R+r)^2, где R - расстояние от центра Земли до тела, r - радиус Земли, m - масса тела, G - константа. Пусть твоя ЗС в N раз меньше Луны по линейным размерам, но видна под тем же углом. Для простоты возьмем плотность равной Луне. Тогда перепад гравитации будет (L для Луны) равен: G (mL / N^3) (RL / N) r / (RL/N-r)^2 (RL/N+r)^2. Если R >> r видно, что и в числитель и в знаменатель N входит в -4 степени и "сократится". (Там настоящего сокращения быть не может, но порядок один).

На пальцах: если ЗС в 10 раз линейно меньше Луны, но в десять раз ближе, она будет тянуть в 10 раз слабее, но за счет того, что отношение "расстояние до объекта / радиус Земли" станет в 10 раз меньше, перепад гравитации будет примерно такой же. Аналогично если она в 10 раз больше Луны, но в десять раз дальше.

Короче, приливы-отливы будут примерно такие же как от Луны, правда с поправкой на плотность твоей ебалы.
412 488376
>>8372
Спасибо.
413 488404
>>8372
>>8376
Кстати, я немного ошибся. Для такого же видимого угла расстояние будет чуток больше, потому что в N раз уменьшится расстояние до поверхности Земли, а не до ее центра. Но принципиально это на ответ не повлияет, может приливы и будут, скажем, на 50% меньше (при N = 10), но по крайней мере того же порядка. Ну и чем ближе N к единице, тем меньше ошибка.

Алсо, пока никто не доебался: камушек в вытянутой руке не дает таких приливов как Луна по двум причинам: одна указана выше, а вторая в оригинальном посте: приближение работает только при R >> r, то есть надо хотя бы R = 3r.
414 488545
Почему пельмени всплывают при варке?
415 488553
>>8545
Надуваются паром и всплывают. Если дать остыть вместе с бульоном - утонут нахой, т.к. пар сконденсируется и пельмешы уменьшутся в объёме, их плавучесть снизится.
416 488564
>>8553
Какое давление поддерживают на космических станциях? Одна атмосфера или меньше? Можно ли там сварить пельмени в кастрюльке-центрифуге или нужна скороварка?
417 488569
>>8564

> Одна атмосфера или меньше?


это

>Можно ли там сварить пельмени в кастрюльке-центрифуге или нужна скороварка?


можно без центрифуги, только крышку плотнее прижимай и вытяжку включить не забудь, парует сильно
418 488571
>>8569

>это


Так одна? Или 0,7, скажем, для экономии?

>только крышку плотнее прижимай


Это уже скороварка будет. Имеется в виду, достаточно ли давления на МКС для традиционной варки в обычной кастрюле если просто создать эрзац-гравитацию, или нужно еще дополнительное давление в скороварке? Дома я могу пельмени в кастрюльке сварить. А на вершине Эвереста, например, вода кипит при 70 градусах, этого вроде как мало для нормальной варки.
419 488578
>>8571

> Так одна? Или 0,7, скажем, для экономии?


одна, раньше бывало заливали меньше, но чистого кислода

> Это уже скороварка будет. Имеется в виду, достаточно ли давления на МКС для традиционной варки в обычной кастрюле если просто создать эрзац-гравитацию, или нужно еще дополнительное давление в скороварке? Дома я могу пельмени в кастрюльке сварить. А на вершине Эвереста, например, вода кипит при 70 градусах, этого вроде как мало для нормальной варки


посыл ясен, довления хватит, но космонавтикам будет ниоч от варки
объём-то закрытый, бздунов не берут, потеть только на тренажорах, вся хуйня
тебя с твоей кастрюлькой недопоймут
420 488582
>>8578
Спасибо. Так-то понятно, что никто там пельмешки варить не будет, мне просто интересна принципиальная возможность.
421 488607
Сколько космонавт нахватается радиации за полет на Марс? Лучше свинца и йода ничего не придумали, а лететь от 4 до 7 месяцев в зависимости от расположения планет.
Читал, что даже на МКС нихуевая доза радиации за год пребывания, а они там под магнитным пледиком Земли
IMG20190611191839679.jpg223 Кб, 900x1017
422 488608
IMG20190611192233068.jpg238 Кб, 900x1017
423 488611
>>8607
не то приклеил)
424 488614
>>8607

> Лучше свинца и йода ничего не придумали


так-то альфу похуй чем гасить лишь бы масса
425 488616
>>8614
Да я хз какая именно радиация в космосе
Ну и корабль свинцом не обшить, дя
426 488618
>>8616
бутерброд между баками с топливом/окислителем/водой/воздухом
платина же
427 488622
>>8611
Пиздец масштаб уебищный. Нихуя не понятно сколько получишь за 180 дней полета до Марса
428 488623
>>8622
логарифмический, жри давай
429 488625
>>7675

>Потому что окислитель


И чо?
>>7704
А диапазон изменения температур ты считал?
>>7876

>если образуется пар, то теплопередача через него сразу резко падает, и стенка может перегреться и прогореть.


Давай, расскажи это паровым котлам.
430 488630
>>8625

> Давай, расскажи это паровым котлам.


> рд


> паровые котлы

431 488633
Можно ли при нынешних технологиях погулять по Титану?

Скажем, условия такие: титанавта в изготовленном Илоном Маском на заказ скафандре (вес на Земле не более 50 кг, чтобы исключить "скафандры"-танки) магически телепортируют на поверхность Титана, в сухое ровное место. То есть, лететь туда не надо. Он должен пешочком протопать минимум 10 км, и пробыть там минимум 3 часа. После чего он так же магически телепортируется обратно. Выживет? Маску дается три года на подготовку.

Условия на поверхности: температура около -180, плотная атмосфера (в 4 раза плотнее земной), давление около 1,5 атм. Ветер слабый, не более 0,5 м/с. Возможна моросня их жидкого метана, но дождя не будет.
432 488635
>>8633

>вес на Земле не более 50 кг


ты охуел, даже скафандры для космоса и луны больше весят, а там вакуум и теплопередача крайне мала
433 488639
>>8635
Ну хорошо, пусть 100 кг.
434 488642
>>8639
Ладно, я сегодня добрый. Пусть 105 вес любой, лишь бы титанавт в нем самостоятельно ходить мог (моторы и прочие экзоскелеты недопускаются). Впрочем, сила тяжести на Титане около 1/7 земной, это должно помочь.
435 488750
>>8241
C-14. А продукты деления урана находятся внутри оболочек ТВЭЛов. Даже при их разрушении и попадании продуктов деления в КМПЦ, они не осядут на графит, потому что СУЗ охлаждаются своей отдельной петлей.
436 488762
Теоретически, что будет, если Ио упадёт на Юпитер?
437 488764
>>8762
Ио сломается.
438 488793
>>8764
Это понятно, а остальное? Что с юпитером будет и остальной солнечной системой?
439 488798
>>8793
похуй вообще
резонанс спутников юпитера немного поменяется и всё
sage 440 488814
>>8630
Продолжай деградировать
image.png898 Кб, 900x777
441 488875
Можно ли увеличить дельту пердежом?

Смотрите, у нас есть корабль до марса и астронавты которые пердят. Можно ли собирать метан при рециркуляции воздуха и использовать его как топливо для дополнительного малого двигателя.
один человек выпускает в день до 0.5 литров газов из которых четверть это метан. То есть примерно 0.125. Четыре астронавта напердят 0.5 метана в день. Если полет продлится 100 дней в одну сторону, то это 50 литров метана. В обе стороны 100 литров. 100 литров это ровно одна бочка.

Внимание вопрос - сколько дельты можно получить из бочки сгоревшего метана?
442 488878
>>8875
Вопрос не имеет смысла без массы корабля, но в любом случае даже метр в секунду вряд ли наберется.
443 488879
>>8875

>из которых четверть это метан


Что-то много. Там же вроде чуть ли не все - воздух, просто побольше азота, поменьше кислорода.
image.jpeg136 Кб, 476x497
444 488880
Платина, наверное.

Поясните за то, как из сверхновой звезды образуется черная дыра.
Ведь при взрыве сверхновой идет выброс массы из ядра наружу, то есть, сверхновая теряет массу, а потом образует черную дыру, которая за счет своей большой массы притягивает все окружающие объекты.

Сча поясню, почему я не понимаю, как из сверхновой образуется ЧД.
Предположим, есть масса Nчд, набирая которую объект получает огромную силу притяжения и становится ЧД.

Но как сверхновая (тут ее масса меньше необходимой Nчд) теряя массу, выбрасывая ее из ядра, становится черной дырой?
Поясните, где я не прав.
Я далек от астрономии, просто увидел ролик на ютубе и решил спросить у вас.
445 488881
>>8880

>Предположим, есть масса Nчд, набирая которую объект получает огромную силу притяжения и становится ЧД.


Лучше не предполагай, это неверно. ЧД может быть любой массы.

ЧД, которая получается из звезды, всегда заметно легче самой звезды, потому что в процессе значительная часть массы разбрасывается взрывом. Масса сворачивается в ЧД не столько потому, что ее много, сколько потому что ее много в малом объеме.

Любое количество вещества стремится занять "нулевой" объем под действием самогравитации. Но ему мешают всяческие силы. В звезде, например, это давление разогретого газа. Вот когда эти силы становятся слишком слабыми в силу разных причин, и не могут компенсировать самогравитацию, вещество коллапсирует и получается ЧД. Другой вариант - вещество сжимают внешними силами, но это не про звезды.
446 488883
>>8881
Теперь понятно, что дело в объеме. Совсем не подумал об этом. Спасибо.

> занять "нулевой" объем


Это очень маленькое межатомное расстояние?
447 488884
>>8883
И даже дальше. В нейтронной звезде, например, уже нет атомов (кроме, может, тонкой внешней "корочки"), их "давит" самогравитацией. Но и там еще есть силы, которые не дают сжаться еще больше - это уже чисто квантовомеханические эффекты.
448 488887
>>8883
Это буквально нулевой, т.е. точка или кольцо нулевой толщины. В сингулярности все элементарные частицы занимают одно и то же положение в пространстве, потому что не существует никаких сил, способных их друг от друга оттолкнуть.
449 488888
Можно ли создать такие бактерии, которые бы хорошо выживали на марсе (экватор), питались бы пылью и чем попало вокруг, атмосферой, солнечной радиацией, и в ходе жизнедеятельности выделяли кислород?
Если заселить из на марсе и дать расти 100 лет, то могут ли они сделать его пригодным для жизни.

Заранее спасибо. Наверняка это невозможно, хотелось бы узнать почему.
450 488889
>>8888
Всей земной жизни нужна жидкая вода, на Марсе ее нет (1-2 крошечных полоски, даже не факт, что являющихся водой — не в счет).

Все остальные факторы типа холода, низкого давления, недостатка химических источников питания, и т.д. сами по себе не непреодолимы, но без жидкой воды жизни 100% не будет.
451 488890
>>8887

>не существует никаких сил, способных их друг от друга оттолкнуть


Никаких известных. Хуй его знает, что там происходит на самом деле. Теоретически предсказать нельзя, ведь ОТО и квантмех не стыкуются, можно лишь делать более-менее обоснованные предположения. А экспериментальных данных тоже нет, поскольку необходимые энергии пока недостижимы, а уж экспериментировать с экстремальной гравитацией вообще непонятно как.
452 488985
Чому больше никто не делает астрономические зарисовки. Ведь это так няшно.
453 489036
Предположим, нам дали пульт управления солнечной системой. И мы без задней мысли немного подкорректировали орбиты всякой мелкой шушеры и ебанули по Марсу десятью тысячами комет диаметром около 10 км каждая, в основном из льда (75% и больше) в течение 10 лет. Насколько это повлияет на Марс: атмосферу, поверхность? Сколько понадобиться времени, чтобы он "успокоился" после такого и там можно было выращивать картофель высаживать экспедиции?
454 489040
>>9036
А если у тебя пульт управления солнечной системой есть, то хули не передвинуть Марс поближе к Солнцу?

Вообще ну ебнул ты кометами, они создали кратеры, газ высвободился, наполнил атмосферу. Знаешь что дальше будет с этим газом? Газ этот сделает финт ушами и пойдет на полярную шапку, осядет там, сделав её толще размером. И как ты ему запретишь? Скажешь - "Эй, газ, а ну стоят на месте, не двигаться и не замерзать, ты должен быть газообразным, не переходи в твёрдое состояние!!!"
Думаешь он тебя будет слушаться? Даже если ты там как ЭЛОН Маск ебнешь ядерными бомбами по полярным шапкам, ты думаешь газ скажет "О нет, полюса разбомбили, я здесь больше не могу затвердевать, место радиоактивное!" Лол, нет. Газ растает, поциркулирует и снова осядет на полярные шапки, как ни в чём не бывало. Что ему помешает блядь?
455 489042
>>9040
Херово. Для парникового эффекта от водяного пара, как я понимаю, жалких 10 тысяч комет недостаточно будет, да? А можно ли в принципе Марс разогреть таким образом (хули, будем считать запас комет бесконечным)? Есть же еще небольшой единовременный разогрев просто от падающих камней, а потом держать полученный профит парником. Двигать орбиту Марса владелец пульта не разрешил, только кометами кидаться.
456 489044
>>9042
Вот тут есть немного с цифрами и расчетами
https://www.youtube.com/watch?v=a6igkZV1qzk
457 489045
>>9044
Спасибо, но 70 минут как-то многовато для меня.
458 489046
>>9042
Там многое зависит от размера комет. Магически ты конечно можешь двигать 10 км кометы, но если бы дело зашло до реальных дел, то тут мы понимаем, что даже 10 метровый камешек на околоземной орбите двинуть для нас невозможно. А уж комета с размерами 10 км и вытянутой орбитой это просто невозможно. Ты там банально охуеешь просто догнать её, а уж чтобы повлиять на эту глыбу это нужно - ну я не знаю, тут просто нереально, чувак тебе просто сложно представить сколько весят эти 10 км камня и какие скорости необходимо применить чтобы изменить орбиту этого колосса. В видео Егоров говорит о том, что нужно под 600 комет размерами с комету Хейлла-Боппа(50-60 км) для того чтобы давление на Марсе стало 1% от земного. В общем просто какие-то диких размеров кометы нужно найти, потом как-то их двигать. 50 км камень прилетающий из Облака Оорта, ему вообще наверное похуй что ты ему не делай, он не заметит даже если ты несколько ракет Сатурн-5 к нему приделаешь.
459 489049
>>9046
Не, я понимаю, что это нереально, отсюда и магический пульт. 50-60 км это где-то в 150-200 раз больше чем 10 км по объему. То есть 600 таких будут примерно в 10 раз существеннее 10 тысяч камушков из вопроса. Другое дело, что даже 1% давления (что уже ~100 000 десятикилометровых комет) это ни о чем, да, никакого парника там не получится.
460 489084
>>9036
Венеробояре ссут в ротеш марсохолопам итт.
461 489085
>>9036
Universe sandbox2 тебе в помощь.
image.jpeg197 Кб, 604x604
462 489096
>>7150

> На таких телах и гравитация будет мизерной, пукнешь и улетишь нахуй.

463 489106
>>9036
высаживать картоху и сажать экспедиции можно хоть сейчас, все условия прекрасно привозятся с землюхи, один хуй ты везти что-то я смотрю собираешься
бомбить итт совершенно не зачем, один хуй приличной атмосферы не сделаешь, а красивые виды цвета разноцветного поноса издырявишь
алсо этого >>9040 не слушай, на марсе уже мульёны лет потепление хуярит, так что не всё в шапки уйдёт, скорее именно полярные головные уборы съебнутся быстро и решительна
464 489122
Насколько жоская в космосе радиация? Интересуют все места: около звезд, всяких разных планет, подальше от всего фонящего. Как там вообще в межзвездном пространстве, чего ожидает современная наука? А в межгалактическом? Какой там тип радиации (альфа-бета-гамма, или какие еще частицы бывают)?
465 489152
>>9122
Гамма лучи, бета излучения и альфа волны. Пиздецома, гроб , кладбище, рогозин
88989065873299.jpg29 Кб, 550x412
466 489164
>>9122
В радиационных поясах планет носятся ионы и электроны по магнитным линиям, это по сути естественный ускоритель частиц. Бахаются о корабль, проникают на некую глубину, вызывают гамму. Радиационные пояса у Юпитера с его мощной магнитосферы имеют такую силу, что проще в ядро реактора сразу залезть.

В межпланетном пространстве тоже летают заряженные частицы - солнце хуячит высокоэнергетичными протонами, во время вспышек особенно. Кроме того, небесный рандом иногда присылает ТЗЧ - тяжелые заряженные частицы, которые ебашат с такой силой, что пробивают десять с половиной аллахов навылет, порождая в стороны дождь из других заряженных частиц по траектории прохождения сквозь материал. От таких защиты нет в принципе.

>Как там вообще в межзвездном пространстве


Примерно как в межпланетном, только попроще, нет солнечного ветра.
467 489170
Как так Аполлоны успешно летали? Такая сложная миссия, столько этапов. И все без ошибок и аварий(13 не считаем).
Так все предусмотрели и рассчитали?
468 489174
>>9170
Так аварии, ошибки, сбои были, дохуя, просто там не такие серъезные. Ну погорели чутка на Аполлон-1. Зато потом были почти никем не вспоминаемые Аполлон-2, 3, 4, 5, 6, 7, 8, 9, 10 во время которых тестили все системы и улучшали, искореняли ошибки. Просто история программы Аполлон это очень длинная история, это как биография человека, которую всем лень читать. Она очень подробна, но сука пиздец людям как лень, не хотят и всё тут. Просто зацикливаются на Аполлон-11 и моменте посадки, а всё что было до, и что было после - все это люди просто игнорируют. Ну и плюс люди не любят читать технические детали миссий, это же тебе не Властелин Колец, не Звездные Войны под попкорн жрать, быдлу это вообще не интересно, там аббревиатуры, фамилии, цифры часто много слишком всего непонятного.
469 489208
>>9164
А ТЗЧ эти из реликтового излучения или нет? В межгалактическом они тоже будут?
470 489228
>>9208

> заряженные частицы


> микроволновый фон


учу читать в ттв, спешите видеть
471 489229
>>9208
ТЗЧ может быть откуда угодно - где-то на другой звезды вспышка произошла, частицы вылетели и полетели. Нейтронные звезды срут там, квазары из других галактик, ебанутые конфигурации из пар звёзд, экзотические всякие объекты могут посылать частицы во всех направлениях, тонны дерьма в нашей галактике и за пределами
472 489230
изображение.png372 Кб, 1080x859
473 489248
Сап, беру подержанный телескоп (Долбоеб). Сразу говорю, я до этого оптику никакую не покупал и астрономией интересуюсь скорее на уровне "ОООО ЕБАТЬ, ЗВЕЗДЫ, КРУТО!" и хотел бы просто попробовать вести наблюдения.
Написал продавцу - пока молчит, вот что о телескопе было в объявлении.
На что мне смотреть и обращать внимание при покупке? Инб4 на засранность линз и на причину продажи, а еще на то, как телескоп хранился
474 489253
Яйцеголовые, почему чистая вода не прозрачная? Почему воду видно в воздушной среде? Почему я вижу пар в виде дыма?
475 489260
>>9228
Но ведь у нас квантово-волновой дуализм.
151446880316642882.jpg20 Кб, 473x269
476 489261
>>9260

>Но ведь у нас квантово-волновой дуализм.


Чево блядь?
477 489263
>>9253

>почему чистая вода не прозрачная?


А она должна быть? У неё свой параметр преломления.
478 489264
>>9261
Корпускулярно-волновой, простити.
479 489281
>>9263

>преломления


Прозрачность. Именно как физическая характеристика. Коэф. преломления это ответ на второй вопрос. А вообще это спейсач.
>>9264

>Корпускулярно-волновой


И к чему ты это. Значение знаешь? Пит буль.
480 489282
>>9281
Ну тип все частицы в какой-то степени одновременно и волны. Могу теперь предположить (по твоей реакции), что реликтовое излучение состоит только из фотонов, но я не знал / не задумывался, честное пионерское!
481 489284
>>9282

>Ну тип все частицы в какой-то степени одновременно и волны.


Ну тип в начале прошлого века ты был бы прогрессивным, но это уже на той же полке, где и атом Бора. Сейчас частицы уже не принято пытаться описывать упругими шариками, которые иногда волны.

>предположить


Смелее предполагай, как-то неуверенно. Да, из фотонов.
482 489285
>>9284
Охуеть ты короче популяризатор науки. Лучше не задавать тупых вопросов в треде тупых вопросов, а то набигут такие буки как ты.
483 489288
>>9229
Это типа такого? Их мало, но они очень суровы?
https://en.wikipedia.org/wiki/Oh-My-God_particle
484 489289
>>9285
Ну для старта - не популяризатор, даже не стремящийся.
А как относиться, когда анон приходит с воззрениями, которым уже сто лет (ну пусть 50-60, когда там ктп Фейнман замесил?), и это его текущий уровень. Ну а самому всерьез отвечать, круглая ли Земля, вертится ли, было бы в радость? Я может тоже зашел что-то новое узнать. Лучше бы не дулся, а почитал что-нибудь популяризаторское, видосики там образовательные посмотрел. Это тот уровень, который можно без обсуждений и вопросов воспринимать, а вот немного освоившись в современных представлениях станет интереснее и обсуждать в тредах тупых вопросов нюансы и странности.
485 489297
>>9288
ну ты так-то совсем уж йобу принёс, но в целом верно, в наши спокойные ебеня такой хуйни особо не прилетает, а если и прилетает, то не задерживется
другое дело совать жопу в квазар или дождаться когда по тебе прицельно (или не очень) жахнет лучем или сверхновой
алсо возле особо крупных и магнино-могучих юпов тоже собирается много тяжелого дерьма( пускай и не настолько адового) к юпам лучше не лезть
monkey confused izryando ozadachen.jpg79 Кб, 500x499
486 489299
>>9288

> At this speed, if a photon were travelling with the particle, it would take over 215,000 years for the photon to gain a 1 cm lead as seen in Earth's reference frame.


переведите плз
487 489300
>>9299

>На этой скорости, если бы фотон летел рядом с частицей, потребовалось бы 215 000 лет, чтобы фотон обогнал ее на 1 см, в системе отсчета Земли.

488 489301
>>9300

>более 215 000

489 489319
>>9248
На состояние зеркал
490 489334
>>9248
Не в Томске покупаешь?
491 489341
>>8183
Не только из-за вреда для костей.
Если внимательно посмотреть как выглядят астронавты на МКС, то станет заметным совершенно иное распределение давлений в организме в условиях без гравитации: чем ближе к сердцу - тем выше давление, чем дальше от сердца - тем меньше давление и меньше кровоток. Лицо, голова и грудная клетка в невесомости припухшие, ноги и нижняя часть туловиша болкк тонкие и "усохшие" по сравнению с наличием силы тяжести.
Поэтому полного лекарственного решения проблемы не существует, нужно либо вращать шконки вместе с огромным тороидом, либо развивать постоянное ускорение от 0.5g и выше.
Но тороид по сравнению с шаром обладает довольно херовым отношением внешней площади к полезному объёму, внешнюю защиту шара можно сделать более толстой.
492 489354
>>9288

>https://en.wikipedia.org/wiki/Oh-My-God_particle


Что будет с космонавтом, если в него попадет такая ебала? Или как вот в этом >>9164 случае, это несмертельно?
493 489359
Анон, у меня вдруг появилась мысль, которая теперь не дает мне покоя: что, если бы у Земли было две Луны?
Это сильно ли бы повлияло на геологию, климат, гравитацию, длительность суток и года, физиологию, развитие ночных видов животных и растений, развитие навигации, астрономии, календаря, земледелия?
Что было было?

Для простоты возьмем два тела совокупной массой равной массе настоящей луны, один объект в 40% от ее массы, второй - 60%.

Приветствуется ссылка на литературу по теме, если такая вдруг окажется.
494 489363
>>9341
Можно не делать огромный тороид, а например, сделать противовес на длинном тросе, вынеся тем самым центр вращения за пределы жилого отсека.

А там уж жилой модуль можно любой формы хуячить.
495 489364
А есть ли снимки Луны на фоне Млечного Пути?
496 489365
>>9354
душ из мульёна частиц опаснее одной еболы (при равной энергии) потому как одна частица потеряет меньше энергии( и соответственно меньше передаст в тело космонавтика)
это верно в случае достаточной для преодоления пути до тела энергии
497 489366
>>9364
Только композитные, в один снимок такое никак не сделать — слишком большая разница в яркости.
498 489367
>>9359
Повлияло бы на некоторые вещи тобою описанные и не повлияло бы на другие, но описывать как именно это повлияло бы на 12 аспектов мироустройства при недостаточных данных сложно да и лень вообще. Нужно знать орбиту второй луны.
499 489368
>>9365
Ну как бы частица может попасть в скафандр и за стенкой скафандра сразу же будет этот душ. Или же она может пройти под углом снизу вверх в тело космонавта, попасть в ногу, прореагировать с кожей и душ из частиц начнет распространяться внутри тела.
500 489369
>>9366

> Только композитные


а если снимать когда на луне ночь?
501 489370
А планет на фоне Млечного Пути? Юпитера там или Сатурна
gc8y3ga4ja331.jpg660 Кб, 4016x2677
502 489371
>>9369
Даже небольшого серпа достаточно для засветки.

>>9370
Ну этого как говна, но они ведь как точки выглядят без сильного увеличения, крутых картинок не выйдет. И все равно разница в яркости настолько большая, что планеты выходят засвеченными добела.
503 489373
>>9367
а какие данные нужны?
504 489374
>>9368
дело в том, что более тяжелые частицы меньше хотят в неупругие столкновения и взаимодействуют главным образом с электронными оболочками
505 489375
>>9374
Всё равно нельзя исключать столкновения - это тебе не нейтрон или нейтрино, из космоса может прилететь ион железа на гигавате чтобы разьебать кому-то сраку.
506 489376
>>9375
половинный пробег ГЭв-ого протона в люминии - полтора метра, так что все высокоебучие говна с КРАЙНЕ ВЫСОКОЙ вероятностью пролетят тельце на вылет практически без последствий для последнего
507 489378
>>9365
То есть 20-50 см воды (грубо говоря, в зависимости от угла попадания) той ебале не хватило бы, чтобы реализовать весь свой "потенциал", и "обломки" соударений все равно вылетели бы на еще огромной скорости?
508 489379
>>9376
Постой, я тут читаю и вижу такую штуку как ионизационные потери, то есть частица высоких энергий может взаимодействовать даже без столкновения, ионизируя атомы по пути следования. Так что даже пролетая навылет через тушку, она может нанести генетический урон, если я правильно понял конечно.
509 489383
>>9378
вообще говоря шансы выпадения "дождя" на таком коротком и неплотном участке крайне малы
и даже в таком случае, ливень не сразу делает из одной частицы мульёны, сперва частица распадётся скажем на 3-5 частиц примерно такой же общей энергии, которые примерно также нихуя не хотят распадаться здесь и сейчас, которые пролетят ещё некоторое расстояние и вероятнее всего покинут обсуждаемое тело космонавтика, толком ни на что не распавшись и особо не тормозя
510 489384
>>9383
Ну да, я примерно это и имел в виду говоря о "потенциале". Типа что есть еще энергия "на поделиться", но уже мишень закончилась, не с чем сталкиваться.
511 489385
>>9379
верно, ты понял правильно, я ведь написал

> взаимодействуют главным образом с электронными оболочками


другое дело что это всё идёт по прямой, имеет околонулевой объем и даёт такие же (никакие) повреждения
512 489387
>>9371
Спасибо, всё равно нормально очень даже. А что это? Юпитер? Спутников не видать? А есть по две планеты?
513 489388
>>9385

>даёт такие же (никакие) повреждения


Нет, это даёт повреждение. В ганимед-треде приводил статью, где риск галактических космических лучей называется главной опасностью при полётах в дальний космос. Причем галактический фон растёт с увеличением расстояния от Солнца и в минимумы солнечной активности.
514 489390
>>9387
Юпитер. Для спутников увеличение побольше нужно, тут все в одно пятно сливается.
516 489418
>>9388
я тут про хуйни, которые дают ливни, а не про "нормальные" космические лучи распинаюсь если чо
а вот про нормальные галактик рэи всё довольно спекулятивно (в плане точности, сам факт что космические лучи опаснее лучей солнышка и отсутствие приемлимых массогабаритных решений никто не оспаривет), потому как спектр довольно широкий и часть уйдёт в титанолюминий обшивки, часть зайдёт, а часть пролетит
считать конкретные доли и величины ионизационных нагрузок без конкретных шипов - такое себе
517 489421
>>9418
Да считай там пизда от галактических лучей в любом случае - от слабых тебе в любом случае нужно защититься при помощи слоёв защиты, а сильные только сдетонируют от этой защиты и создадут ливень.
Тред утонул или удален.
Это копия, сохраненная 17 сентября 2019 года.

Скачать тред: только с превью, с превью и прикрепленными файлами.
Второй вариант может долго скачиваться. Файлы будут только в живых или недавно утонувших тредах. Подробнее

Если вам полезен архив М.Двача, пожертвуйте на оплату сервера.
« /spc/В начало тредаВеб-версияНастройки
/a//b//mu//s//vg/Все доски